You are on page 1of 198

HAN 3010, HUMAN ANATOMY PAST PAPERS

The GIANT
2021/08/17 Giant compilation

Table of contents
2015 Test 2 p3 to 10
2015 Test 2 marking key p11 to 18
2017 test 1. P19 to 39
Other Han questions p40 to 56
Random questions and answers p57 to 65
2021 test 1 preps tutorial material with Simon Hatimbula, #Hats MD. p66 to 88
2017 assessment one p89 to 97
Examination past papers pages 99 to198
 Synthesis of fats an lipoproteins in the liver
 Absorption, synthesis, and transport of fats in small intestines
 Synthesis and storage of glycogen in liver and skeletal muscle
78. Name phases of interphase in correct order , giving major activity of each phase
 G1-normal metabolic cell activity
 S-phase- DNA replication
 G2- protein synthesis and centriole duplication, storage of energy needed for cell
division.
 G0- resting phase
79. Corpus Luteum; a hormone secreting glandular structure formed after ovulation at the site of
ruptured follicle
80. GLANDS
 MEROCRINE------salivary glands
 Holocrine-----sebaceous glands
 Apocrine ----mammary gland
81. GLANDS
 Simple acinar- paraurethral, sebaceous gland
 Compound branched tubular-duodenal, bartholins glands.
 Unicellular- goblet cell
 Compound acinar- mammary glands
 Simple coiled tubular-sweat glands
 Compound tubule acinar- salivary, lacrimal glands
82. EPITHELIUM
 Cornea – stratified squamous non keratinised
 Conjunctiva-stratified columnar
 Gall bladder- simple columnar
 Lips- stratified squamous epithelium keratinised
 Uterine tubes-ciliated simple columnar epithelium
 Vagina- stratified squamous epithelium non keratinised
 Epididymis-sterio ciliated pseudo stratified columnar epithelium
 Trachea- ciliated pseudo stratified columnar epithelium
 Ureter- transitional epithelium
 Renal tubules-simple cuboidal
 Pleura-simple squamous epithelium
 Palm of hand-squamous epithelium keratinised
 Oesophagus-stratified squamous epithelium non keratinised
 Salivary duct-simple cuboidal epithelium
 Bulburethral gland – simple cuboidal epithelium

 Internal aspect of blood vessels-simple squamous epithelium
83. HEMIDESMOSOMES; cells linked to basal membrane
84. Only one osteocyte is found in each lacunae
SCHOOL OF MEDICINE

DEPARTMENT OF ANATOMY

HUMAN ANATOMY CONTINUOUS ASSESSMENT TEST 11

Date: 16th June 2015 Computer Number:………………………………

Time: 2 hours Maximum Marks:

SECTION I : SHORT ANSWER QUESTIONS

1. Define: (20)
i) Vermilion border
……………………………………………………………………………………………………………………………………
……………………………………………………………………………………………………………………………………
…………………………………………………………………………………………………………………………………..
ii) Sinusoid
……………………………………………………………………………………………………………………………………
……………………………………………………………………………………………………………………………………
……………………………………………………………………………………………………………………………………
iii) Supra-nuclei cap
……………………………………………………………………………………………………………………………………
……………………………………………………………………………………………………………………………………
……………………………………………………………………………………………………………………………………
iv) Krause end bulbs
……………………………………………………………………………………………………………………………………
……………………………………………………………………………………………………………………………………
…………………………………………………………………………………………………………………………………...
v) Centroacinar cells
……………………………………………………………………………………………………………………………………
……………………………………………………………………………………………………………………………………
……………………………………………………………………………………………………………………………………
vi) Superfetation
……………………………………………………………………………………………………………………………………
……………………………………………………………………………………………………………………………………
……………………………………………………………………………………………………………………………………
vii) Gastrulation
……………………………………………………………………………………………………………………………………
……………………………………………………………………………………………………………………………………
……………………………………………………………………………………………………………………………………
viii) Hatching of the blastocyst
……………………………………………………………………………………………………………………………………
……………………………………………………………………………………………………………………………………
……………………………………………………………………………………………………………………………………
ix) Syncytiotrophoblast
……………………………………………………………………………………………………………………………………
……………………………………………………………………………………………………………………………………
……………………………………………………………………………………………………………………………………
x) Venous Portal System
……………………………………………………………………………………………………………………………………
……………………………………………………………………………………………………………………………………
……………………………………………………………………………………………………………………………………

2. List the types of cells that line liver sinusoids and their functions (6)
…………………………………………………………………………………………………………………………………………………
…………………………………………………………………………………………………………………………………………………
…………………………………………………………………………………………………………………………………………………
…………………………………………………………………………………………………………………………………………………

3. Name the parts of the adrenal cortex and what each part secretes (6)
…………………………………………………………………………………………………………………………………………………
…………………………………………………………………………………………………………………………………………………
…………………………………………………………………………………………………………………………………………………
………………………………………………………………………………………………………………………………………………..

4. Name the blood vessels that supply the anterior abdominal wall (5)
…………………………………………………………………………………………………………………………………………………
…………………………………………………………………………………………………………………………………………………
…………………………………………………………………………………………………………………………………………………
…………………………………………………………………………………………………………………………………………………
…………………………………………………………………………………………………………………………………………………
………………………………………………………………………………………………………………………………………………..

5. Name the three structures that pierce the clavipectoral fascia (3)
…………………………………………………………………………………………………………………………………………………
…………………………………………………………………………………………………………………………………………………
…………………………………………………………………………………………………………………………………………………
…………………………………………………………………………………………………………………………………………………

6. Name the type of joint the proximal radioulnar joint is, and the ligament that holds the head
of the radius to the ulnar (2)
…………………………………………………………………………………………………………………………………………………
…………………………………………………………………………………………………………………………………………………
…………………………………………………………………………………………………………………………………………………

7. Give the boundaries and contents of the cubital fossa (8)


…………………………………………………………………………………………………………………………………………………
…………………………………………………………………………………………………………………………………………………
…………………………………………………………………………………………………………………………………………………
…………………………………………………………………………………………………………………………………………………
…………………………………………………………………………………………………………………………………………………
…………………………………………………………………………………………………………………………………………………
…………………………………………………………………………………………………………………………………………………
………………………………………………………………………………………………………………………………………………..

8. Name the muscles that form the thenar eminence and give their nerve supply (4)
…………………………………………………………………………………………………………………………………………………
…………………………………………………………………………………………………………………………………………………
…………………………………………………………………………………………………………………………………………………
…………………………………………………………………………………………………………………………………………………
…………………………………………………………………………………………………………………………………………………

9. Give the components of the pilosebecious unit (3)


…………………………………………………………………………………………………………………………………………………
…………………………………………………………………………………………………………………………………………………
…………………………………………………………………………………………………………………………………………………
………………………………………………………………………………………………………………………………………………..

10. List the lymphoid organs that have no germinal centers (2)
…………………………………………………………………………………………………………………………………………………
…………………………………………………………………………………………………………………………………………………
…………………………………………………………………………………………………………………………………………………
…………………………………………………………………………………………………………………………………………………

11. List the components of the juxtaglomerular apparatus (3)


…………………………………………………………………………………………………………………………………………………
…………………………………………………………………………………………………………………………………………………
…………………………………………………………………………………………………………………………………………………
………………………………………………………………………………………………………………………………………………..

12. Name the histological layers of the wall of the thoracic oesophagus in order of their
arrangement beginning from the luminal aspect (4)
…………………………………………………………………………………………………………………………………………………
…………………………………………………………………………………………………………………………………………………
…………………………………………………………………………………………………………………………………………………
…………………………………………………………………………………………………………………………………………………
…………………………………………………………………………………………………………………………………………………
………………………………………………………………………………………………………………………………………………..
13. Name the types of intercellular junctions found in intercalated discs and where exactly in
intercalated discs they are found (6)
…………………………………………………………………………………………………………………………………………………
…………………………………………………………………………………………………………………………………………………
…………………………………………………………………………………………………………………………………………………
…………………………………………………………………………………………………………………………………………………
…………………………………………………………………………………………………………………………………………………

14. List the coverings of the testes, in the testes in, their order from the outside to the innermost
(9)
…………………………………………………………………………………………………………………………………………………
…………………………………………………………………………………………………………………………………………………
…………………………………………………………………………………………………………………………………………………
…………………………………………………………………………………………………………………………………………………
…………………………………………………………………………………………………………………………………………………
…………………………………………………………………………………………………………………………………………………
…………………………………………………………………………………………………………………………………………………
…………………………………………………………………………………………………………………………………………………

15. Name the five stages of fertilization (5)


…………………………………………………………………………………………………………………………………………………
…………………………………………………………………………………………………………………………………………………
…………………………………………………………………………………………………………………………………………………
…………………………………………………………………………………………………………………………………………………
…………………………………………………………………………………………………………………………………………………
…………………………………………………………………………………………………………………………………………………

16. List the results of fertilization (5)


…………………………………………………………………………………………………………………………………………………
…………………………………………………………………………………………………………………………………………………
…………………………………………………………………………………………………………………………………………………
…………………………………………………………………………………………………………………………………………………
…………………………………………………………………………………………………………………………………………………
…………………………………………………………………………………………………………………………………………………

17. List the main features of the embryo at two weeks (8)
…………………………………………………………………………………………………………………………………………………
…………………………………………………………………………………………………………………………………………………
…………………………………………………………………………………………………………………………………………………
…………………………………………………………………………………………………………………………………………………
………………………………………………………………………………………………………………………………………………...

18. What causes cranio-caudal folding (2)


…………………………………………………………………………………………………………………………………………………
………………………………………………………………………………………………………………………………………………..
19. What causes lateral folding (2)
…………………………………………………………………………………………………………………………………………………
……………………………………………………………………………………………………………………………………………….

20. List the cell types of the interstial tissue of the testes (3)
…………………………………………………………………………………………………………………………………………………
…………………………………………………………………………………………………………………………………………………
…………………………………………………………………………………………………………………………………………………
………………………………………………………………………………………………………………………………………………..

21. Name the histological layers of the uterine walls (3)


…………………………………………………………………………………………………………………………………………………
…………………………………………………………………………………………………………………………………………………
…………………………………………………………………………………………………………………………………………………
…………………………………………………………………………………………………………………………………………………

SECTION II

Label the diagrams below

1.
2.

3.
4.

5.
6.

END OF TEST
SCHOOL OF MEDICINE

DEPARTMENT OF ANATOMY

HUMAN ANATOMY CONTINUOUS ASSESSMENT TEST 11

Date: 16th June 2015 Computer Number:………………………………

Time: 2 hours Maximum Marks:

SECTION I : SHORT ANSWER QUESTIONS

1. Define: (20)
i) Vermilion border
It’s the transition point on the lip were the SSE keratinised changes to SSS non
keratinised
ii) Sinusoid
These are capillaries with a wider lumen diameter than others, partial discontinuous
basement membranes and large fenestra in the endothelial cells and poor junctions
iii) Supra-nuclei cap
……………………………………………………………………………………………………………………………………
……………………………………………………………………………………………………………………………………
……………………………………………………………………………………………………………………………………
iv) Krause end bulbs
……………………………………………………………………………………………………………………………………
……………………………………………………………………………………………………………………………………
…………………………………………………………………………………………………………………………………...
v) Centroacinar cells
These are spindle shaped cells that insert into the acinar lumen and secrete the
bicarbonate to neutralise the chyme
vi) Superfetation/superfecundation
The occurrence of a second conception during pregnancy, giving rise to embryos of
different ages
vii) Gastrulation
……………………………………………………………………………………………………………………………………
……………………………………………………………………………………………………………………………………
……………………………………………………………………………………………………………………………………
viii) Hatching of the blastocyst
……………………………………………………………………………………………………………………………………
……………………………………………………………………………………………………………………………………
……………………………………………………………………………………………………………………………………

ix) Syncytiotrophoblast
……………………………………………………………………………………………………………………………………
……………………………………………………………………………………………………………………………………
……………………………………………………………………………………………………………………………………
x) Venous Portal System
This is a system of veins inbetween two capillary beds (hepatic portal system,
hypothalamichypophyseal portal system, renal (arterial) and splenic-)
2. List the types of cells that line liver sinusoids and their functions (6)
1. Ito/stallete cell- store vitamin A
2. Cholangiocytes –for regeneration
3. Kupfer cell- phagocytosis
4. Hepatocytes- parenchymal cells
3. Name the parts of the adrenal cortex and what each part secretes (6)
1. Zona glomerulosa- mineralocorticoids eg aldosterone
2. Zona fasculata- glucocorticoids eg cortisol
3. Zona reticularis- weak androgens- dihydroepiandrosterone
4. Name the blood vessels that supply the anterior abdominal wall (5)
1. Epigastric arteries
2. Branch from musculophrenic artery
3. Superficial circumflex iliac artery
4. Deep circumflex iliac artery
5. Inferior epigastric artery
6. Superior epigastric
5. Name the three structures that pierce the clavipectoral fascia (3)
…………………………………………………………………………………………………………………………………………………
…………………………………………………………………………………………………………………………………………………
…………………………………………………………………………………………………………………………………………………
…………………………………………………………………………………………………………………………………………………

6. Name the type of joint the proximal radioulnar joint is, and the ligament that holds the head
of the radius to the ulnar (2)
…………………………………………………………………………………………………………………………………………………
…………………………………………………………………………………………………………………………………………………
…………………………………………………………………………………………………………………………………………………

7. Give the boundaries and contents of the cubital fossa (8)


…………………………………………………………………………………………………………………………………………………
…………………………………………………………………………………………………………………………………………………
…………………………………………………………………………………………………………………………………………………
…………………………………………………………………………………………………………………………………………………
…………………………………………………………………………………………………………………………………………………
…………………………………………………………………………………………………………………………………………………
…………………………………………………………………………………………………………………………………………………
………………………………………………………………………………………………………………………………………………..

8. Name the muscles that form the thenar eminence and give their nerve supply (4)
…………………………………………………………………………………………………………………………………………………
…………………………………………………………………………………………………………………………………………………
…………………………………………………………………………………………………………………………………………………
…………………………………………………………………………………………………………………………………………………
…………………………………………………………………………………………………………………………………………………

9. Give the components of the pilosebecious unit (3)


…………………………………………………………………………………………………………………………………………………
…………………………………………………………………………………………………………………………………………………
…………………………………………………………………………………………………………………………………………………
………………………………………………………………………………………………………………………………………………..

10. List the lymphoid organs that have no germinal centers (2)
…………………………………………………………………………………………………………………………………………………
…………………………………………………………………………………………………………………………………………………
…………………………………………………………………………………………………………………………………………………
…………………………………………………………………………………………………………………………………………………

11. List the components of the juxtaglomerular apparatus (3)


1. JuxtaGlomerula cells
2. Macular densa
3. Extraglomerula mesangial cells (polkissen/lasis)
12. Name the histological layers of the wall of the thoracic oesophagus in order of their
arrangement beginning from the luminal aspect (4)
1. Mucosa
2. Muscularis (interna) mucosae
3. Submucosa
4. Muscularis (extenar)

13. Name the types of intercellular junctions found in intercalated discs and where exactly in
intercalated discs they are found (6)
1. Gap junctions – basal lateral surface
2. Desmosomes – Apical lateral surface
3. Fascia adherens – basal aspect of the cell

14. List the coverings of the testes, in the testes in, their order from the outside to the innermost
(9)
1. Skin of the scrotum
2. Dartos muscle
3. External spermatic fascia
4. Cremasteric fascia and muscle
5. Internal spermatic fascia
6. Parietal Tunica vaginalis
7. Visceral Tunica vaginalis
8. Tunica albuginea
9. Tunica vasculosa
15. Name the five stages of fertilization (5)
…………………………………………………………………………………………………………………………………………………
…………………………………………………………………………………………………………………………………………………
…………………………………………………………………………………………………………………………………………………
…………………………………………………………………………………………………………………………………………………
…………………………………………………………………………………………………………………………………………………
…………………………………………………………………………………………………………………………………………………

16. List the results of fertilization (5)


…………………………………………………………………………………………………………………………………………………
…………………………………………………………………………………………………………………………………………………
…………………………………………………………………………………………………………………………………………………
…………………………………………………………………………………………………………………………………………………
…………………………………………………………………………………………………………………………………………………
…………………………………………………………………………………………………………………………………………………

17. List the main features of the embryo at two weeks (8)
…………………………………………………………………………………………………………………………………………………
…………………………………………………………………………………………………………………………………………………
…………………………………………………………………………………………………………………………………………………
…………………………………………………………………………………………………………………………………………………
………………………………………………………………………………………………………………………………………………...

18. What causes cranio-caudal folding


The cranio-caudal folding is due the rapid growth of the notochord and central nervous
(neurotube) system (2)
19. What causes lateral folding (2)
Rapidly growing spinal cord and somites
20. List the cell types of the interstial tissue of the testes (3)
1. Leydig cells
2. Fibroblasts
3. Myoid cells contractile cells
21. Name the histological layers of the uterine walls (3)
1. Endometrium
2. Myometrium
3. Perimetrium

SECTION II

Label the diagrams below

1.
G- interlobular artery
H-PCT
I-DCT
E-Renal capsule
F-glomerulas
D- Interlobular vein
J- vasa recta
B-thin descending limb of the loop of henle
C- thin ascending limb of henle
A-Collecting duct

2.
3.

1. Anterior rectus sheath


2. Anterior cutaneous branch of the 11th intercostal nerve
3. Inguinal ligament
4. Spermatic cord
5. Anterior cutaneous branch of the ilioinguinal
6. Anterior cutaneous branch of the iliohypogastric
7. External oblique muscle and its aponeurosis
8. Rectus abdominis muscle
9. Linea alba

4.

5.
6.

1. Median arcuate ligament


2. Subcostal nerve
3. Psoas major
4. Iliohypogastric
5. Ilioinguinal
6. Genital femoral nerve
7. Iliucus muscle
8. Lateral cutaneous nerve of the thigh
9. Femoral nerve
10. Auricular surface of the sacrum
11. Obturator nerve
12. Right cruz of the diaphragm
END OF TEST
SCHOOL OF MEDICINE

DEPARTMENT OF ANATOMY

ANATOMY CONTINUOUS ASSESSMENT I

Date: 20th January, 2017 Computer No:…………………………

Time: 2 Hours Total Marks: 130

Answer All Questions

SECTION I: MULTIPLE CHOICE QUESTIONS Marks: 120

Choose one most appropriate answer from the alternatives A-D in each question by
marking an X in the answer grid sheet. There is no penalty for wrong answers.

1. Which of the following is correct concerning basket cells?


A. Are stimulated by hormones
B. Similar with smooth muscle cells in origin
C. Surround the basal lamina
D. Are scattered among secretory cells

2. …………………………… line is seen as a line of demarcation between lighter flexor


and the darker extensor parts of the skin of limbs.
A. Futcher’s
B. Wrinkle
C. Occupational
D. Contour

3. Regarding the resting skin tension lines, which statement is correct?


A. Are inappropriate for surgical incision
B. Pinching at right angle will produce more tension lines
C. Are altered by contraction of muscles deep to them
D. Are same as kraissl lines

4. Skin creases limited to the cornified (uppermost layer of the epidermis) layer are
termed…………………………..
A. Primary
B. Secondary
C. Tertiary
D. Quaternary

1
5. Brunner’s glands are an example of …………………………………….
A. Simple acinar
B. Simple branched acinar
C. Compound branched tubular
D. Compound tubulo-acinar

6. What type of a gland is the structure in the picture below.

A. Tubulo-acinar
B. Mucous
C. Serous
D. Sero-mucous

7. The boundaries of the mediastinum include the following, except……………………


A. Sternal angle
B. Diaphragm
C. Thoracic vertebrae
D. First pair of ribs

8. Which of the following statements correctly describes the mediastinum?


A. It is divided into superior and inferior mediastinum by the sternal angle
B. The posterior wall of the pericardium forms the posterior wall of the posterior
mediastinum
C. The heart and great vessels are the contents of the anterior mediastinum
D. The pericardial cavity is found in the middle mediastinum

2
9. Which of the following statements is true about the pericardium?
A. The serous pericardial reflection surrounding the aorta and pulmonary trunk is
anterior to the oblique sinus
B. The anterior wall of the oblique sinus is the serous pericardium of the posterior wall
of the left atrium
C. The transverse sinus is anterior to the inferior vena cava, superior vena cava and the
pulmonary veins
D. A hand passed inferior to the apex of the heart, then posterior superiorly will end up
in the transverse sinus

10. Which of the following valves will produce the ‘lub’ sound when closing?

A. 1 and 2
B. 2 and 3
C. 1 and 4
D. 3 and 4

11. The following statements about blood vessels are true, except………………………..
A. Superficial veins communicate with deep veins via anastomotic veins
B. The radial vein is a deep vein
C. Collateral circulation gives an alternative route of blood supply to an area
D. The cephalic and brachial veins can be connected by a perforating vein

3
12. Which of the following structures has been appropriately matched?
A. Great Saphenous vein deep vein
B. Left common carotid artery originate from brachiocephalic
trunk
C. Pulmonary artery comes from right ventricle
D. Right Subclavian vein drains in the right Brachiocephalic vein

13. Which of the following statements about the lymphatic vessels is incorrect?
A. The thoracic duct is the largest lymphatic duct in the body
B. The cysterna chyli receives lymph from the intestinal lymphatic vessel
C. The right lumbar lymph vessel drains into the right lymphatic duct
D. The right bronchomediastinal lymph vessel drains lymph into the thoracic duct

14. Which of the following is NOT true concerning mitochondria?


A. They replicate independently from the cell
B. They are passed on via the sperm
C. Contain their own genetic material
D. Vary in morphology between cells

15. Which of the following organelles in liver cells help detoxify alcohol after taking some
dry spirits (tujirijiri)?
A. Smooth endoplasmic reticulum
B. Golgi apparatus
C. Lysosomes
D. Nucleus

16. Which of the following organelles will be abundant in fibroblasts?


A. Free ribosomes
B. Proteasomes
C. Rough endoplasmic reticulum
D. Golgi apparatus

17. ……………………………… are numerous in highly metabolic cells.


A. Ribosomes
B. Smooth endoplasmic reticulum
C. Proteasomes
D. Mitochondria

18. Which of the following is false concerning ribosomes


A. Are basophilic
B. They are manufactured in the nucleolus
C. Are called Nissil bodies in neurons
D. They synthesize lysosomes

4
19. Which of the following statements is true about cells
A. All are surrounded by a cell wall
B. Animal cells are eukaryotic
C. The nucleus is not membrane –bound in human cells
D. Ribosomes and the nucleus are the two basic parts of our cells

20. Which statement is true about the plasma membrane?


A. It is made up of a single layer of phospholipids
B. The phospholipids have nonpolar heads
C. The phospholipids have polar fatty acid tails
D. The phospholipid molecules are amphipathic

21. Which of the following organelles are responsible for the formation of the granules in
mast cells?
A. Golgi apparatus
B. Lysosomes
C. Rough endoplasmic reticulum
D. Smooth endoplasmic reticulum

22. ………………………………….degrade unwanted proteins in cells.


A. Proteasomes
B. Smooth endoplasmic reticulum
C. Lysosomes
D. Mitochondria

23. Concerning the cell cycle


A. In G1 phase the cells are resting
B. M phase, cells enter cellular growth
C. G0 phase, cells synthesize DNA
D. G2 phase, cells have a double complement of cellular DNA

24. Secretory cells of sweat glands release their products by exocytosis. They are an
example of…………………………….glands.
A. Holocrine
B. Merocrine
C. Apocrine
D. Mucous

5
25. Which of the following structures is the crista terminalis?

26. In the diagram above (Question 25), which of the following structures supplies blood to
the right upper limb?
A. 1
B. 2
C. 3
D. 4

27. Which of the following structures is the oblique cardiac vein

6
28. Which of the following are inclusion bodies?
A. Mitochondria
B. Glycogen
C. Keratin
D. Lysosomes

29. Which of the following is an example of hyaline cartilage?


A. Intervertebral discs
B. Epiglottis
C. Epiphyses
D. Knee menisci

30. Hyaline cartilage……………………………..


A. forms glenoid labrum
B. does not ossify with age
C. relatively vascular
D. forms epiphyseal growth plates

31. An example of a synovial joint is


A. intervertebral disc
B. sternomanubrial joint
C. sacroiliac joint
D. epiphyses

32. An example of a secondary cartilaginous joint


A. costochondral joint
B. intervertebral disc
C. Temporomandibular joint
D. lambdoid suture

33. What type of joint is the 1st sternocostal joint


A. Secondary cartilaginous
B. Typical synovial
C. Primary cartilaginous
D. Fibrous

34. Which of the following movements are permitted at the joints named
A. Condyloid joint – biaxial
B. Hinge joints- multiaxial
C. Pivot joint – multi axial
D. Saddle joint – multiaxial

35. The following joint is a syndesmosis:


A. Distal tibiofibular
B. Proximal tibiofibular
C. Interphalangeal
D. Humeroulnar

7
36. The following is an example of an elastic cartilage:
A. Costal cartilages
B. Epiglottis
C. Cricoid cartilage
D. Arytenoid cartilage

37. The following are wandering cells of connective tissue except:


A. Mast cells
B. Basophils
C. Eosinophils
D. Neutrophils

38. Mast cells are


A. Epithelial
B. Produce antibodies
C. Phagocytic
D. Produce SRA

39. Plasma cells have


A. Eccentrically placed nuclei
B. Well-developed endoplasmic reticulum
C. Phagocytic ability
D. Basophilic cytoplasm

40. Mucous connective tissue.


A. Is present in adult
B. Is typical of umbilical cord
C. Found in nostrils
D. Typical of vaginal secretions

41. Tendons are;


A. Dense regular connective tissue
B. Rich in elastic fibers
C. Are rich in reticular fibers
D. Has deficiency in collagen fibers

42. Articular cartilage ;


A. Is covered by perichondrium
B. Is elastic cartilage
C. Easily regenerate following injury
D. Receive nutrition via diffusion from synovial fluid

8
43. The matrix of hyaline cartilage contains;
A. Nerves
B. Capillaries
C. Chondroitin- 4 –sulfate
D. Elastic fibers

44. Fibrobasts
A. Phagocytic
B. Produce antibodies
C. Syntheses collagen
D. Are mature cells of connective tissue

45. Collagen fibers are;


A. Basophilic
B. Acidophilic
C. Metachromatic
D. Birefrigent

46. Reticular fibers are;


A. Readily seen with H and E staining
B. Argyrophilic
C. PAS positive
D. Contains elastin

47. Aponeuroses are ;


A. Found in central nervous system
B. Found in the peripheral nervous system
C. flattened sheets of dense irregular connective tissue
D. composed of collagenous bundles

48. Brown fat cells are;


A. Unilocular
B. Found in fetuses
C. Under hormonal regulation
D. More abundant than white fat in adults

49. The sternal angle of Louis is an important anatomical landmark on the sternum that can
be used to count ribs. What is its vertebral level?
A. C3/C4
B. C5/C6
C. T2/T3
D. T4/T5

9
50. Which of the following statements is correct about the nervi erigentes?
A. Consists of somatic fibres from sacral spinal nerves S2, S3, S4
B. Consists of postganglionic fibres from sacral spinal nerves S2, S3, S4
C. Responsible for motor supply of the detrusor muscle
D. Supplies the external urethral sphincter

51. The following is a parasympathetic activity:


A. Tachycardia
B. Miosis
C. Mydriasis
D. Bronchodilatation

52. Which part of the lower end of the humerus articulates with the ulna?
A. Capitulum
B. Trochlea
C. Lateral epicondyle
D. Medial condyle

53. Which of the following is correct about the gray rami communicantes?
A. Consists of preganglionic parasympathetic fibres
B. Nerve fibres are largely unmyelinated
C. Carries preganglionic sympathetic fibres
D. Fibres arise from the anterior horn cells of the spinal cord

54. Which vertebral level is safe to perform a lumbar puncture in a neonate?


A. L3/L4
B. L1/L2
C. L4/L5
D. L5/S1

55. The following are components of the brain stem:


A. Medulla oblongata, diencephalon, midbrain
B. Telencephalon, midbrain, pons, cerebellum
C. Medulla oblongata, cerebellum, pons
D. Medulla oblongata, pons, midbrain

56. Which of the following bones is the first to ossify in utero:


A. Talus
B. Femur
C. Clavicle
D. Hyoid

10
57. Eversion and inversion of the foot takes place at which joint?
A. Ankle joint
B. Calcaneocuboid joint
C. Subtalar joint
D. Femorotibial joint

58. Which of the following nerves is likely to be injured in surgical fracture of the humerus?
A. Radial nerve
B. Axillary nerve
C. Median nerve
D. Musculocutaneous nerve

59. Which of the following joints is most commonly dislocated?


A. Hip joint
B. Glenohumeral joint
C. Temporomandibular joint
D. Radiocarpal joint

60. The dermatome at the umbilicus is……


A. T8
B. T9
C. T10
D. T11

61. Which of the following is not part of the pelvic inlet?


A. Promontory of the sacrum
B. Arcuate line of the ilium
C. Pecten pubis
D. Iliac crests

62. Which of the following cranial nerves originate from the midbrain?
A. Olfactory
B. Oculomotor
C. Abducent
D. Facial

63. Which of the following is a plane joint:


A. Distal tibiofibular
B. Interphalangeal
C. Glenohumeral
D. Stenoclavicular

11
64. Which parasympathetic ganglion is responsible for the supply of the lacrimal gland?
A. Stellate
B. Ciliary
C. Otic
D. Sphenopalatine

65. Which of the following cranial nerve is purely sensory?


A. Vestibulocochlear
B. Glossopharyngeal
C. Trigeminal
D. Trochlear

66. At what level is the inferior angle of the scapula?


A. C6
B. T7
C. T10
D. L1

67. Pronation and supination of the forearm takes place at which joint?
A. Humero-ulnar joint
B. Humeroradial joint
C. Proximal radioulnar joint
D. Radiocarpal joint

68. Which nerve runs in close proximity to the medial epicondyle of the humerus?
A. Radial nerve
B. Median nerve
C. Musculocutaneous nerve
D. Ulnar nerve

69. Which part of the lower end of the humerus commonly fractures in children?
A. Supracondylar
B. Medial epicondyle
C. Lateral epicondyle
D. Trochlea

70. The pisiform bone is a sesamoid bone in a tendon of which of the following muscle?
A. Extensor carpi ulnaris
B. Flexor pollicis longus
C. Flexor carpi ulnaris
D. Extensor digiti minini

12
71. Which of the following carpal bones is commonly fractured?
A. Hamate
B. Capitate
C. Trapezoid
D. Scaphoid

72. Which of the following pelvic bone prominences is important in determination of the
fetal station during labour?
A. Ischial tuberosities
B. Pubic crests
C. Iliac crests
D. Ischial spines

73. At which point is the femur commonly fractured in old age?


A. Neck
B. Trochanteric
C. Subtrochanteric
D. Diaphyseal

74. The following cranial nerves have parasympathetic outflow:


A. Olfactory, accessory
B. Facial, glossopharyngeal
C. Abducent, trochlear
D. Vestibulocochlear, optic

75. Which parasympathetic ganglion is responsible for accommodation of the pupil?


A. Stellate
B. Ciliary
C. Otic
D. Submandibular

76. Which of the following is true about splanchnic nerves?


A. Responsible for supplying the pupillary muscles
B. Fibres pass via prevertebral ganglia minus synapsing
C. The greater splanchnic nerves are from T10-T12
D. Have motor innervation to skeletal muscles

77. Which of the following statements is not true of the typical female pelvis?
A. Has longer/prominent ischial spines
B. Pubic angle is obtuse
C. The inter-tuberosity distance is able to admit at least 4 knuckles
D. The pelvic inlet is heart-shaped

13
78. Which of the following is not TRUE about tissues of the human body?
A. Nervous tissue has no extracellular matrix
B. Epithelial barriers are not important components of innate immunity
C. Connective tissue has abundant extracellular matrix
D. Junctional complexes are also present in muscle tissue

79. Which of the following is true about an axoneme?


A. Dynein is present/functional in Kartagener’s syndrome
B. Made up of actin filaments
C. Non-motile cilia has a 9+0 pattern
D. Found in the core of only cilia and not flagella

80. Choose the TRUE statement about the location of junctional complexes:
A. Macula adherens are found virtually in all mammalian tissues
B. Desmosomes when found in nervous tissue are called electrical synapses
C. Intermediate junctions in cardiac muscle are called fascia adherens
D. Gap junctions are restricted to the basal part of the lateral membranes

81. Choose the incorrect answer about epithelial tissue:


A. Cells continuously renew by mitosis
B. Has tight junctions which anchor cells of the epithelium together
C. Cells have structural and functional polarisation
D. Basal laminae may serve as passageway for cell migration

82. Which of the following is not an epithelial cell surface specialisations?


A. Stereocilia
B. Cilia
C. Villi
D. Microvilli

83. Which epithelium lines the paranasal air sinuses?


A. Simple squamous
B. Simple columnar
C. Ciliated pseudostratified columnar
D. Non-keratinized stratified squamous

84. Which of the following is true about change in epithelium?


A. Dysplasia is a physiological change
B. Change of epithelium in lower oesophagus in a patient with gastroesophageal
reflux disease (GERD) to simple columnar (Barrett’s oesophagus) is metaplastic
C. Change of epithelium in respiratory tract of a heavy smoker to stratified squamous
(non-keratinized) is dysplastic
D. Disruption of change of epithelium to non-keratinized stratified squamous in the
endocervix is dysplastic

14
85. Which of the following is not TRUE about functions of epithelial tissue?
A. Sensation is well illustrated in the olfactory neuroepithelium
B. Absorption is well illustrated by cilia present in lining of small intestines
C. Contractility is exhibited by the myoepithelial cells
D. Secretion is well illustrated in glandular epithelia

86. The following cells belong to the mononuclear phagocytic system EXCEPT:
A. Dust cells
B. Osteoclasts
C. Mast cells
D. Langerhans cells

87. Which of the following statements is true?


A. Basophils are also known as pus cells
B. The nuclear-cytoplasmic ratio is higher in monocytes compared to lymphocytes
C. Lymphocytes are usually seen with an indented C-shaped nucleus
D. In bacterial infections, Neutrophils tend to decrease in blood.

88. Which of the following are the most abundant cells at the site of acute inflammation?
A. Macrophages
B. Lymphocytes
C. Monocytes
D. Neutrophils

89. Which of the following cells produce antibodies?


A. B-lymphocytes
B. T-lymphocytes
C. Plasma cells
D. Monocytes

90. Which of the following is TRUE about bone cells?


A. Osteoblasts are located at the surfaces of bone tissue arranged like a stratified
epithelium
B. Like cartilage, each lacuna maybe occupied by multiple cells
C. Osteocytes occupy lacunae and their processes of adjacent cells make contact with
each other via harvesian canals
D. A ruffled border is formed on active osteoclast surface facing bone matrix

91. The normal range for white blood cell (WBC) count is:
A. 12-15 x 10^12/L
B. 4-5 x 10^12/L
C. 150-400 x 10^12/L
D. 4-11 x 10^9/L

15
92. The average life span for thrombocytes is:
A. 120 days
B. 10 days
C. 20 days
D. 100 days

93. Which of the following statements is not TRUE about erythropoiesis?


A. Howell-Jolly bodies are acidophilic nuclear remnants of DNA in erythrocytes
B. An orthochromatophilic erythroblast has less basophilia compared to
polychromatophilic erythroblast
C. Haemoglobin concentration increases from proerythroblast to erythrocyte stage
D. A basophilic erythroblast has a larger nuclear volume and less condensed chromatin
compared to an orthochromatophilic erythroblast

94. Choose the incorrect statement about lymphocytes below:


A. B and T-lymphocytes are indistinguishable in routine blood staining
B. Helper T-lymphocytes have cluster of differentiation 4 (CD4)
C. B-lymphocytes differentiate into plasma cells
D. Final differentiation and maturation of T-lymphocytes happens in the thyroid gland

95. All the following are myeloid cells EXCEPT:


A. Erythrocytes
B. Memory T-cells
C. Thrombocytes
D. Polymorphonuclear leukocytes (PMLs)

96. Which of the following statements is not TRUE about the bone marrow?
A. The newborn has no yellow marrow
B. Haematopoietic stem cells are toti-potential
C. The endothelium of the blood capillaries in the bone marrow is discontinuous
D. Bone marrow stroma is mostly fibroblasts and delicate reticular fibres

97. Defects involving either a qualitative or quantitative reduction in collagen type 1 fibres
can lead to brittle bone disease. This disease is also called:
A. Osteogenesis imperfecta
B. Osteitis fibrosa cystica
C. Osteopetrosis
D. Osteomalacia

16
98. Which of the following is not TRUE about hormones that regulate calcium in the body?
A. Parathyroid hormone (PTH) is released when there is hypocalcaemia so that it
stimulates bone resorption
B. PTH is essential for life
C. Osteoblasts have receptors for PTH
D. Calcitonin is secreted by the follicular cells of the thyroid gland

99. Which of the following statements is FALSE about woven bone?


A. Is characterised by irregular arrangement of type 1 collagen
B. Is the first bone to appear in fracture repair
C. Has a lower proportion of osteocytes than lamellar bone
D. Has higher mineral content compared to lamellar bone

100. Choose the incorrect statement about the periosteum and endosteum:
A. the periosteum is double-layered
B. Both are important in provision of nourishment to the bone tissue
C. Sharpey fibres bind the endosteum to the bone
D. Provides bone to grow by appositional growth

101. Which of the following stain blue with H&E stain?


A. Cytoplasm
B. Collagen fibers
C. Nucleus
D. Elastic fibers

102. What is Wright's stain used primarily for?


A. Blood
B. Fat
C. Nervous tissue
D. Elastic fibers

103. What color do elastic fibers stain with Verhoeff Elastic stain?
A. Red/Orange
B. Pink/red
C. Purple/Red
D. Blue/black

104. During the preparation of a routine H&E slide, what step occurs after the tissue is
preserved?
A. Fixation
B. Embedding in paraffin
C. Staining
D. Dehydration

17
105. During the preparation of a routine H&E slide, what allows the tissue to be
visualized?
A. Fixation
B. Embedding in paraffin
C. Staining
D. Slicing

106. Which of the following would be best suited to visualize reticular fibers?
A. Masson's trichrome stain
B. Hematoxylin and eosin stain
C. Sudan stain
D. Silver impregnation

107. Which hormone causes the uterus to increase glycogen?


A. progesterone
B. FSH
C. LH
D. Estrogen

108. Which of the following is not a source of estrogen?


A. adrenal gland
B. ovary
C. pituitary
D. Follicle

109.The outer serosal layer of the uterus is the _____.


A. perimetrium
B. cervix
C. endometrium
D. myometrium

110. What is the first structure to receive the oocyte at ovulation?


A. fimbriae
B. ampulla
C. infundibulum
E. isthmus

111. How many oocytes are there in the ovaries of an adult?


A. fewer than 400
B. 10 million
C. 50 million
D. 400,000

112. Which cells are present in a male age five?


A. spermatocytes
B. spermatozoa
C. Sertoli cells
D. Spermatids

18
113. Concerning the skeletal muscle
A. all muscles cross one or more joints
B. many actions are automatic
C. the number of muscle fibers is not dependent on the shape
D. the name of the muscle trapezius refers to its function

114. Concerning skeletal muscle


A. nerves usually enter the superficial surface of the muscle
B. a motor point is the same as a motor unit
C. denervated muscle is able to contract
D. atrophy of muscle is seen only in denervation

115. The following are all methods of testing muscles except:


A. palpation
B. dissection
C. electromyography
D. rigor mortis

116. Concerning tendons and aponeuroses


A. tendons are supplied by sensory nerves
B. tendons have a good blood suppy
C. tendons can not be cut and transplanted
D. aponeuroses tend to be thick and round

117. Bursae
A. are filled with serum
B. may be present where skin moves over a bone
C. do not communicate with joint cavites
D. are seen in superficial fascia

118. Which of the following statement is correct.


A. in eccentric action of a muscle the force of contaction is equal to the load
B. the term teres refers to fast acting muscle
C. the anatomical insertion is the most mobile part of attachment
D. tone in completely relaxed muscle is due to passive elastic tension

119. In the ovarian cycle,


A. Development starts off from cells in the second meiotic division
B. Primodial follicle is surrounded by cuboidal cells
C. The corpus luteum secrets only progesterone
D. Its under the stimulation of FSH

120. In fertilization,
A. Acrosin for penetration of zona pellucida
B. To prevent polyspermia, the corona radiator does not disperse
C. The sperm head is the only part that enters the oocyte
D. The oocyte is in prophase

19
SECTION II Marks: 10

Write T for True and F for false against each statement(score correct answer- 1
mark, wrong answer- -1/2, blank- 0).

121. Individual muscle are usually described by their blood supply

122. A study of a paralyzed muscle can give valuable information on its function

123. A muscle cannot be stretched beyond a certain point without injury is known as
active insufficiency

124. Synergists are prime movers crossing two or more joint

125. Condenser lens is important in resolution

126. Electromicroscopes utilise confocal lens

127. Vesalius is regarded as the father of medicine

128. Capacitation occurs in the uterine tubes

129. In spermiogenesis the second meiotic division is completed F

130. Trophoblast layer contributes to the development of the placenta T

END OF TEST

20
Scanned by CamScanner
Scanned by CamScanner
Scanned by CamScanner
Scanned by CamScanner
Scanned by CamScanner
Scanned by CamScanner
Scanned by CamScanner
Scanned by CamScanner
Scanned by CamScanner
Scanned by CamScanner
Scanned by CamScanner
Scanned by CamScanner
Scanned by CamScanner
Scanned by CamScanner
Scanned by CamScanner
Scanned by CamScanner
Scanned by CamScanner
QUESTIIONS AND ANSWERS

ANATOMY

1. The visceral layer /internal layer of the Bowman’s capsule contains podocytes-the modified
epithelium
2. Transitional epithelium in the renal/kidneys in found in the renal pelvis, calyces, bladder and
ureter.
3. Vibrissae are thick short hairs found in the nasal cavity which filter out large particles from
inspired air.
4. Explain Goose flesh: when the contraction of the erector pili muscle promotes an erection of the
hair shaft into a more vertical position in human beings.
5. PERICYTES: are cell of mesenchymal origin which surround the endothelial cells (contain
myosin, actin and tropomyosin) and have co-ntractile function and proliferate and differentiate
into blood vessels, connective tissue cells and participate in repair process after tissue damage
6. Billroth cords: a compact appearance of macrophages and lymphatic cells in the red pulp of the
spleen.
7. Lymphatic nodules are found in the connective tissues, mucosa, (MALT), lymph nodes, spleen,
tonsils BUT NOT IN THE THYMUS.

8. Tissue grafts and organ transplants are classified as autografts when the transplanted
tissues or organs are taken from the individual receiving them, isografts when taken
from an identical twin, homografts or allografts when taken from an individual (related
or unrelated) of the same species, and heterografts or xenografts when taken from an
animal of a different species.
9. Name five characteristics of sinusoidal capillaries
 Dilated and tortuous diameter
 Contain multiple fenestrations
 Have no continuous basal lamina
 Discontinuous endothelium
 Contain macrophages
10. Classify arteries
 Large, elastic conducting arteries, aorta, pulmonary artery, subclavian arteries.
 Medium sized, muscular distributing arteries; radial, brachial artery
 Smaller, arterioles and resistance vessels
11. Name three cells of the epidermis and their functions
 Melanocytes- for synthesis of melanin that determines colour of the skin
 Keratinocytes- for synthesis of keratin
 Langerhans cells- capable of binding, processing and presenting antigens to T-
lymphocytes
 Merkel’s cells- found in the thick skin of palms and soles, they are chemoreceptors
12. Name the layer s of the epidermis that produce keratin
 Stratum Basale/Germinativum- contain stem cells responsible for intense mitotic
activity and responsible for renewal of epidermal cells. Have basophilic columnar cells.
 Stratum spinosum- contains tonofilaments which maintain cohesion among cells and
resist the effects of abrasion (soles of feet). Has a malpighian layer. Has cuboidal
polygonal cells
 Stratum granulosum- contain keratohyaline granules rich in proteins
 Stratum lucidum- a translucent layer with extremely flattened eosinophilic epidermal
cells. More pronounced in thick skin. Stage of keratinisation.
 Stratum corneum- consists of 20 layers of nonnucleated keratinised cells whose
cytoplasm is filled with keratin and also has horny cells. Has dead keratinised cells.
13. Malpighian layer is the site for all mitoses and the only layer with epidermal stem cells. Consists
of stratum Basale and spinosum.
14. Malpighian corpuscles are the renal corpuscles consisting of the Bowman’s capsule and the
glomerular
15. Blood-brain barrier consists
 Endothelial cells
 Basement membrane
 Astrocytes
16. List the cells of the respiratory epithelium
 Basal /stem cells
 Brush /neuroendocrine /Kulchisky cells
 Mucous goblet cells
 Small granules cells
 Ciliated columnar cells
17. Clara cells- found in the terminal bronchiole, are devoid of cilia, secrete proteins in their apex
and protects bronchioles against oxidative pollutants and inflammation.
18. Functions of Clara cells
 Trap air borne toxins
 Produce cytochrome
 Secrete surfactant
 Detoxification- prevent emphysema
 Act as stem cells
19. Dust cells- these are alveolar macrophages (lung macrophages) found on the surface of alveoli
whose function is to remove irritants and particles in the alveoli by phagocytosis.
20. Space of Disse- is the sub-endothelial space in the liver that separates the endothelium from the
hepatocytes
21. Glassy Membrane- non cellular hyaline layer formed from the thickening of the basal lamina
which separates the hair follicles from the dermis.
22. Pneumocyte type ii- is responsible for synthesis of surfactant.
23. Pneumocyte type I – is responsible for gas exchange.
24. List the cell of the stomach*
 Enteroendocrine cells
 Chief ( zymogenic ) cells
 Parietal ( oxyntic ) cells; stomach+; secrete hydrochloric acid and intrinsic factor
 Stem cells
 Mucous neck cells
25. During which part of mitosis does the nuclear membrane disappear? Metaphase
26. During which stage of meiosis does the nuclear membrane disappear? Metaphase
27. B (beta)-cells found in pancreas.
28. Name the components of the circulatory system; blood and lymphatic vascular system
29. Brunner’s glands secrete urogastrone.
30. Ciliated Pseudo-stratified columnar epithelium seen in penile urethra, epididymis, respiratory
tract(trachea alae nasae, bronchus, larger bronchioles, vas deferens
31. Sebaceous glands are seen in glans penis, lips, eyelid, areolar, labia minora, nasal cavity,
scrotum, external nose and conjunctiva.
32. Sebaceous glands are examples of holocrine glands
33. What type of epithelium is olfactory epithelium? Pseudo stratified columnar epithelium
34. What cells are found in the olfactory epithelium
 Basal cells
 Bipolar cells
 Supporting cells
35. Name the major salivary gland- all are branched acinar/ alveolar glands
 Parotid glands- branched acinar, secretes serous, pyramidal shaped, and largest
 Submandibular glands- branched tubular acinar, secretes mucous and serous, irregular
 Sublingual glands- branched tubular acinar, secretes mucous and serous, and smallest
36. Name the tonsils found in the body and their location and features
 Palatine tonsils: located in the lateral walls of the oral portion of the pharynx
 They are 2
 Lined with stratified squamous epithelium
 Densely infiltrated with lymphocytes
 Has 10-20 invaginations or crypts that contain live and dead bacteria
and inflammatory cells
 Has a thick capsule that protects against spread of tonsillar infections
 Pharyngeal tonsils: located on the upper posterior portion of the pharynx
 Single tonsil
 Lined with ciliated pseudo stratified columnar epithelium
 Some areas have stratified epithelium
 Has no crypts
 Has diffuse lymphoid tissue and lymphoid nodules
 Has a thin capsule
 Lingual tonsils: located at the base of the tongue
 They are numerous and smallest
 Lined with stratified squamous epithelium
 Each capsule has a crypt
37. What is Waldeyers ring? a collection of palatine, lingual and pharyngeal tonsils
38. Goose Flesh: when the contraction of the erector pili muscles promotes the erection of the hair
follicle into a more vertical position in human beings
39. Classify capillaries
 Continuous/somatic; found in skin, connective, nervous and muscles tissues and
exocrine gland; responsible for transport of macromolecules in both directions; as a
blood brain barrier
 Sinusoid or discontinuous found in the liver, bone marrow and spleen; for
immune/defence due to the presence of macrophages and for enhanced interchange
between blood and tissues.
 Fenestrated in renal tissue and GIT; facilitates active absorption
40. Classification of Ribs
 True ribs; first 1- 7 pairs attached to costal cartilage
 False ribs; 3 pairs (8,9,10) attach to the costal cartilage of the true rib above or 7
 Floating; 2 (11 & 12) suspended ribs below the false ribs; they are not attached to the
cartilage arterially.
41. Post capillary venule function;
 participates in inflammatory processes and
 Exchange of cells and molecules between blood and tissues.
42. Name five regions of the body in which the sebaceous glands secrete directly onto the skin
surface
 Glans penis
 Lips
 Eye lids
 Labia minora
 Clitoris
 areolar
43. Nexus is gap junction area of communication between adjacent cells that allow passage of very
small particles across a small intercellular gap within the junction, found in the heart and
muscles.
44. Polysomes (POLYRIBOSOMES): are individual ribosomes held together by strands of mRNA
45. Payer’s Patches have M cells that trap antigens from the lumen.
46. I cells produced in intestines and secrete CCK
47. Functions of the immune system include
 Opsonisation- facilitates phagocytosis
 Activation of complement system
 Secretes cytokines IL1, IL2, IL6, TNF and interferons
 Mediators of the immune system
48. Brunner’s Glands: these are sub mucosal glands of the duodenum that secrete alkaline mucus
which protects the mucosa of the small intestine from the effects of gastric acid and brings the
intestinal contents to the optimal PH for pancreatic enzyme action. They also secrete
urogastrone which inhibit the secretion of gastric acid. They are also called Duodenal Glands.
49. Macula densa: is a group of tall closely packed distal tubule cells that lie adjacent to the
Juxtaglomerular cells, they play an important role in regulating the rate of filtrate formation
50. Muscularis Mucosae: also known as muscle layer or Muscularis externa. The outer most layer
of the GIT tract composed of the inner oblique, middle circular and outer longitudinal layers. It
has the myenteric or Auerbach’s plexus between the two muscle layers.
51. Components of the blood –air barrier:
 Pneumocyte type I cells
 Basement membrane
 Endothelial cells
52. Name the cell types found in the small intestine epithelium
 Brush/stem cells; differentiate to other cells
 Paneth cells; secrete defensins and lysosomes which inhibit bacteria
 Neuroendocrine cells; secrete VIP, serotonin, Somatostatin
 Goblet cells; secrete mucus
 Enterocytes; are absorptive cells
53. Name the adaptations of the small intestines to improve absorption of food
 It is long
 Have Villi.

 Has crypts of intestinal glands- crypts of lierberkuhn
 Has microvilli- brush border
 Mucosa thrown into circular folds or pricae Muscularis.
54. ITO cells (stellate cells) are found in the liver and are important for vitamin A and D storage.
55. Ameloblast are enamel secreting cells
56. Odontoblasts are predentine secreting cells
57. Cementocytes are cementum secreting cells
58. Bowman’s space or URINARY SPACE is the space lying in the Bowman’s capsule.
59. Renal Capsule is the site for the process of urine formation
60. Name the constituents of the juxtaglomerular apparatus and their functions
 Macula densa- filtrate formation
 Lacis cells (extraglomerular messangial cells)- contain renin
 Juxtaglomerular cells- secrete renin
61. Deficiency of vitamin D leads to OSTEOMALACIA in adults and RICKETTS in children
62. Intramembranous ossification of bones is where bone is laid directly in the fibrous membrane
and examples are; clavicle, vault of skull, mandible
63. Endochondral ossification is where bone ossification is preceded by formation of cartilaginous
model which is later replaced by bone.
64. The four basic human tissue types are
 Connective tissue-support and protection
 Muscle tissue- movement
 Nervous tissue-transmission of nerve impulses
 Epithelia tissue- lining of surfaces or body cavities, glandular secretions.
65. Metaphysis is a portion of the diaphysis adjoining the epiphyseal plate and is active for bone
formation and hence highly vascular.
66. Cartilage contain chondrocytes
67. Types of cartilage
 Elastic cartilage; pinna of ear, epiglottis,
 Hyaline cartilage: costal cartilage joints, articular cartilage, larynx, trachea, nasal septum
 Fibro cartilage; pubic symphysis, vertebrae joints, manubriosterno joints,
temporomandibular joints, sternoclavicular joints,
68. Classification of epithelia
 Squamous epithelium- flattened
 Cuboidal epithelium- height and width same
 Columnar epithelium- height greater than width
69. Functions of the kidneys include
 Production, storage and voiding of urine
o Filtration
o Active absorption
o Passive absorption
o Secretion of waste products
 Maintenance of homeostasis
 Hormonal production
o Erythropoietin
o Renin
70. Name the types of pappillar seen on the human tongue
 Fungiform
 Filiform
 Circumvallate pappillar
71. The term DESMOSOME refers to Macula Adherens or spot junction
72. CEMENTUM: a bone like structure which secures the teeth in its socket
73. Keratinosomes; are epidermal lysosomes which facilitate the adhesion of keratin.
74. Name 8 tissues that do not have lymphatics
 Nails
 Hair
 Tendon
 Cartilage
 Placenta
 Epidermis
 Teeth
 Cornea
 Splenic pulp, bone marrow
75. Name types of neurones and one site where they are found
 Bipolar; Bipolar has one axon and one dendrite- retina, olfactory,
 Unipolar; has a single process
 Multipolar; has one axon and several dendrites- in motor
 Pseudo Unipolar: spinal ganglia, cranial ganglia.
76. PERIKARYON is the same as the cell body or SOMA.
77. Functions of smooth endoplasmic reticulum
 Synthesis of phospholipids and cholesterol
 Synthesis of steroid based hormones e.g. testosterone
 Synthesis of fats an lipoproteins in the liver
 Absorption, synthesis, and transport of fats in small intestines
 Synthesis and storage of glycogen in liver and skeletal muscle
78. Name phases of interphase in correct order , giving major activity of each phase
 G1-normal metabolic cell activity
 S-phase- DNA replication
 G2- protein synthesis and centriole duplication, storage of energy needed for cell
division.
 G0- resting phase
79. Corpus Luteum; a hormone secreting glandular structure formed after ovulation at the site of
ruptured follicle
80. GLANDS
 MEROCRINE------salivary glands
 Holocrine-----sebaceous glands
 Apocrine ----mammary gland
81. GLANDS
 Simple acinar- paraurethral, sebaceous gland
 Compound branched tubular-duodenal, bartholins glands.
 Unicellular- goblet cell
 Compound acinar- mammary glands
 Simple coiled tubular-sweat glands
 Compound tubule acinar- salivary, lacrimal glands
82. EPITHELIUM
 Cornea – stratified squamous non keratinised
 Conjunctiva-stratified columnar
 Gall bladder- simple columnar
 Lips- stratified squamous epithelium keratinised
 Uterine tubes-ciliated simple columnar epithelium
 Vagina- stratified squamous epithelium non keratinised
 Epididymis-sterio ciliated pseudo stratified columnar epithelium
 Trachea- ciliated pseudo stratified columnar epithelium
 Ureter- transitional epithelium
 Renal tubules-simple cuboidal
 Pleura-simple squamous epithelium
 Palm of hand-squamous epithelium keratinised
 Oesophagus-stratified squamous epithelium non keratinised
 Salivary duct-simple cuboidal epithelium
 Bulburethral gland – simple cuboidal epithelium

 Internal aspect of blood vessels-simple squamous epithelium
83. HEMIDESMOSOMES; cells linked to basal membrane
84. Only one osteocyte is found in each lacunae
IMPORTANT: ONLY ATTEMPT THESE QUESTIONS AFTER STUDYING THE NERVOUS SYSTEM
ORGANISATION SLIDES. THESE QUESTIONS SERVE THE PURPOSE OF HELPING YOU PAY
ATTENTION TO DETAIL AS YOU STUDY.

NERVOUS SYSTEM ORGANISATION PART 1

ANSWER TRUE OR FALSE FOR EACH QUESTION. FOR EVERY FALSE ANSWER, GIVE THE
CORRECT ANSWER.

1. Myenteric nerve plexus is in the submucosa layer


2. Auerbach's nerve plexus is in muscularis propria layer
3. Pelvic Splanchnic Nerves also called nervi erigentes, arise from sacral spinal nerves S2,
S3, S4 and consists of postganglionic fibres
4. Pelvic Splanchnic Nerves provide parasympathetic innervation hindgut (from junction
of proximal 1/3rd and distal 2/3rd of the transverse colon to the anus)
5. Trunk of the lesser splanchnic nerve enters the aorticorenal ganglion
6. Least splanchnic nerve runs inferiorly to enter the renal plexus
7. Main trunk of the greater splanchnic nerve enters the inferior aspect of the coeliac
ganglion
8. Roots of the greater splanchnic nerve are from T5 – T10
9. Preganglionic fibres of splanchnic nerves synapse in one of the paravertebral ganglia
10. The 3 major divisions of the abdominal prevertebral plexus and associated ganglia are:
1. Coeliac plexus 2. Aortic plexus 3. Inferior hypogastric plexus
11. Paravertebral ganglia are usually 5 in the cervical region
12. Ganglion impar is anterior to the coccyx
13. Cell bodies of the preganglionic neurons are located in the white matter
14. Parasympathetic trunks (chains) that extend from the base of the skull to the coccyx
where they unite at the ganglion impar
15. Submandibular ganglion is associated with CN VII
16. Sphenopalatine ganglion is associated with CN IX
17. Spenopalatine ganglion is also known as pterygopalatine ganglion
18. Otic ganglion is associated with CN X
19. CN VIII is parasympathetic
20. The ciliary ganglion is associated with the lacrimal gland
21. Otic ganglion is associated with the parotid salivary gland
22. Oculomotor nerve is associated with the ciliary ganglion
23. Dermatome of the nipple is T6
24. Dermatome of the navel is T10
25. A dermatome is area of skin supplied by a single segmental cranial nerve

Prepared by Clive
IMPORTANT: ONLY REFER FROM THIS MARKING KEY AFTER GENUINELY ATTEMPTING THE
QUESTIONS ON NERVOUS SYSTEM ORGANISATION PART I

NERVOUS SYSTEM ORGANISATION PART I

1. F – Myenteric nerve plexus is found in the muscularis propria


2. T
3. F – PSN consist of preganglionic fibres
4. F – Junction is proximal 2/3 and distal 1/3 of transverse colon
5. T
6. T
7. F – Superior aspect
8. F – T5 – T9
9. F -Splanchnic nerves synapse in prevertebral ganglion
10. F – Superior hypogastric plexus
11. F – Usually 3
12. T
13. F – Grey matter
14. F – Sympathetic trunks
15. T
16. F – CNVII
17. T
18. F – CNIX
19. F – Parasympathetic CNs: CNIII, CNVII, CNIX, CNX
20. F – Lacrimal gland is associated with sphenopalatine ganglion
21. T
22. T
23. F – T4
24. T
25. F – single segmental spinal nerve not cranial nerve

MUST KNOW

Parasympathetic cranial nerves and associated ganglion

Definition of dermatome and roots common dermatomes of nipple, umbilicus, perineum

Definition of a splanchnic nerve

Major divisions of the abdominal prevertebral plexus

Functions of both sympathetic and parasympathetic nervous systems in how they effect the
body

Roots of the thoracic splanchnic nerves and associated prevertebral ganglion

Prepared by Clive
Scanned by CamScanner
Scanned by CamScanner
Scanned by CamScanner
Scanned by CamScanner
Scanned by CamScanner
Scanned by CamScanner
Scanned by CamScanner
Scanned by CamScanner
Scanned by CamScanner
Scanned by CamScanner
Scanned by CamScanner
Scanned by CamScanner
Scanned by CamScanner
Scanned by CamScanner
Scanned by CamScanner
Scanned by CamScanner
Scanned by CamScanner
Scanned by CamScanner
Scanned by CamScanner
Scanned by CamScanner
Scanned by CamScanner
Scanned by CamScanner
Scanned by CamScanner
Scanned by CamScanner
Scanned by CamScanner
Scanned by CamScanner
Scanned by CamScanner
Scanned by CamScanner
Scanned by CamScanner
Scanned by CamScanner
Scanned by CamScanner
Scanned by CamScanner
Scanned by CamScanner
EXAM PAST PAPERS FOLLOW NOW
THE UNIVERSITY OF ZAMBIA
UNIVERSITY EXAMINATIONS – JULY 2016
HUMAN ANATOMY
HAN 3010

TIME ALLOWED: 3 HOURS MAXIMUM MARKS:

INSTRUCTIONS TO STUDENTS
1. Attempt ALL questions
2. Read the instructions to each Section.
3. Write your computer number on each paper submitted

COMPUTER NUMBER:_________________________

1
SECTION I: ONE CORRECT ANSWER MULTIPLE CHOICE QUESTIONS 100

Select the one most appropriate answer from the alternatives A – E in each question, by
marking X on the answer sheet grid. There is no penalty for wrong answer

1. During a fight a man is stabbed in the lateral chest beneath the right arm. The wound
does not enter the chest cavity. Physical examination reveals that the vertebral
(medial) border of the patient's scapula projects posteriorly and is closer to the
midline on the injured side. On return visit the patient complains that he cannot reach
as far forward (such as to reach for a door knob) as he could before the injury. The
nerve injured which caused these symptoms is the:

A. axillary
B. long thoracic
C. musculocutaneous
D. radial
E. suprascapular

2. A man suffers a penetrating wound through the anterior axillary fold, with resulting
damage to one of the main terminal branches of the brachial plexus. Among the
effects is a significant weakening of flexion of the elbow. One or more other effects to
be expected is (are):
A. Loss of cutaneous sensation on the tips of several fingers
B. Only loss of cutaneous sensation on the anterolateral surface of the forearm
C. Only weakening of flexion at the shoulder
D. Weakening of flexion at the shoulder and loss of cutaneous sensation on the
anterolateral surface of the arm
E. Weakening of flexion at the shoulder and loss of cutaneous sensation on the
anterolateral surface of the forearm

3. A person sustains a left brachial plexus injury in an auto accident. After initial
recovery the following is observed: 1) the diaphragm functions normally, 2) there is
no winging of the scapula, 3) abduction cannot be initiated, but if the arm is helped
through the first 45 degrees of abduction, the patient can fully abduct the arm. From
this amount of information and your knowledge of the formation of the brachial
plexus where would you expect the injury to be:

A. axillary nerve
B. posterior cord
C. roots of plexus
D. superior trunk
E. suprascapular nerve

4. The cords of the brachial plexus are:


A. above the clavicle, medial to the scalenus medius
B. above the clavicle, anterior to the scalenus anterior
C. below the clavicle, closely related to the axillary artery.
D. below the clavicle, closely related to the subclavian vein
E. four in number

2
5. A man riding a motorcycle hit a wet spot in the road, lost control, and was thrown
from his bike. He landed on the right side of his head and the tip of his shoulder,
bending his head sharply to the left and stretching the right side of his neck.
Subsequent neurological examination revealed that the roots of the 5th and 6th
cervical nerves had been torn away from the spinal cord.

Following the above injury, which of the movements of the arm at the shoulder would
you expect to be totally lost?
A. adduction
B. abduction
C. flexion
D. extension
E. medial rotation

6. Following the above injury there would most likely be diminished cutaneous
sensation over what part of the upper limb?
A. the back of the shoulder
B. the pectoral region
C. the top of the shoulder and the lateral side of the arm
D. the medial side of the arm and forearm
E. the tip of the little finger

7. During an industrial accident, a sheet metal worker lacerates the anterior surface of
his wrist at the junction of his wrist and hand. Examination reveals no loss of hand
function, but the skin on the thumb side of his palm is numb. Branches of which nerve
must have been severed?
A. Lateral cutaneous nerve of the forearm
B. Medial cutaneous nerve of the forearm
C. Median
D. Radial
E. Ulnar

8. The nerve which passes through the quadrangular space of the posterior shoulder
innervates which muscle?
A. Deltoid
B. Infraspinatus
C. Subscapularis
D. Supraspinatus
E. Teres major

9. In the axilla the pectoralis minor is a landmark, and is closely related to all of the
following structures except:
A. cephalic vein
B. cords of the brachial plexus
C. lateral thoracic artery
D. medial pectoral nerve
E. second part of the axillary artery

3
10. The axillary nerve arises directly from which part of the brachial plexus?
A. inferior trunk
B. lateral cord
C. medial cord
D. middle trunk
E. posterior cord

11. Which of the following is not a direct branch of the axillary artery?
A. anterior circumflex humeral
B. posterior circumflex humeral
C. thoracoacromial
D. thoracodorsal
E. subscapular

12. In a fracture of the surgical neck of the humerus, which artery may be injured?
A. Subscapular
B. Posterior humeral circumflex
C. Radial recurrent
D. Deep brachial
E. Circumflex scapular

13. While riding a bike, a patient fell against a tree and fractured the shaft of the humerus
at midlength. What nerve may be injured because of its close proximity to the injury?
A. Ulnar
B. Radial
C. Axillary
D. Medial antebrachial cutaneous
E. Median

14. As an inexperienced phlebotomist (blood drawer) attempts to insert the needle to draw
blood from the median cubital vein, the patient suddenly screams and complains of
pain and burning in the middle and thumb side of his palm. The nerve accidentally
impaled on the needle was the
A. lateral cutaneous nerve of the forearm
B. medial cutaneous nerve of the forearm
C. median
D. ulnar
E. posterior cutaneous nerve of the forearm

15. The general name for an alternate pathway of blood flow in or around an organ,
around a joint, or past an obstruction is called:
A. an arteriovenous anastomosis
B. a periarticular network
C. a perivascular plexus
D. a venous plexus
E. collateral circulation

4
16. When people are severely burned they have great difficulty with fluid loss because the
portion of the skin which is the fluid barrier is destroyed. Which layer of the skin is
responsible for preventing fluid loss?
A. dermis
B. epidermis
C. investing fascia
D. panniculus adiposus
E. subcutaneous tissue

17. Supination of the hand and forearm would be diminished by loss of radial nerve
function. But one very powerful supinator would remain intact and unaffected,
namely:
A. Brachialis
B. Brachioradialis
C. Biceps brachii
D. Flexor carpi radialis
E. Supinator

18. The anterior interosseous is a branch of which nerve?


A. Axillary
B. Median
C. Musculocutaneous
D. Radial
E. Ulnar

19. What muscle is innervated by branches of both the median and ulnar nerves?
A. Flexor carpi ulnaris
B. Flexor digitorum profundus
C. Flexor digitorum superficialis
D. Flexor pollicis longus
E. Pronator quadratus

20. Compression of the median nerve in the carpal tunnel affects which hand muscle?
A. Dorsal interossei
B. Flexor pollicis brevis
C. Flexor pollicis longus
D. Opponens digiti minimi
E. Palmar interossei

21. The pulse of the radial artery at the wrist is felt immediately lateral to which tendon?
A. Abductor pollicis longus
B. Extensor pollicis longus
C. Flexor carpi radialis
D. Flexor digitorum profundus
E. Palmaris longus

5
22. The following description pertains to questions 29 and 30:A patient sustained multiple
deep lacerations on the palm of his hand and anterior surface of his wrist. During
examination, the physician put a piece of paper between adjacent surfaces of the
patient's index and middle fingers and found him unable to squeeze them together
with sufficient force to hold the paper. What muscles are being tested?
A. First dorsal and first palmar interosseous muscles
B. First dorsal and second palmar interosseous muscles
C. First lumbrical and second dorsal interosseous muscles
D. Second dorsal and first palmar interosseous muscles
E. Adductor pollicis

23. The most specific nerve branch to these muscles is the:


A. Deep branch of the ulnar nerve
B. Median nerve
C. Recurrent (motor) branch of the median nerve
D. Superficial branch of the ulnar nerve
E. Ulnar nerve

24. The second costal cartilage can be located by palpating the:


A. costal margin
B. sternal angle
C. sternal notch
D. sternoclavicular joint
E. xiphoid process

25. The sternocostal surface of the heart is formed primarily by the anterior wall of which
heart chamber?
A. Left atrium
B. Left ventricle
C. Right atrium
D. Right ventricle
E. None of the above

26. A patient involved in an automobile accident presents with a sharp object puncture of
the middle of the sternum at about the level of the 4th or 5th costal cartilage. If the
object also penetrated pericardium and heart wall, which heart chamber would most
likely be damaged?
A. Left atrium
B. Left ventricle
C. Right atrium
D. Right ventricle
E. All of the above

6
27. You are caring for a 68-year-old male who has copious amounts of fluid in the left
pleural cavity due to acute pleurisy. When you examine him as he sits up in bed
(trunk upright), where would the fluid tend to accumulate?
A. costodiaphragmatic recess
B. costomediastinal recess
C. cupola
D. hilar reflection
E. middle mediastinum

28. A 23-year-old male injured in an industrial explosion was found to have multiple
small metal fragments in his thoracic cavity. Since the pericardium was torn
inferiorly, the surgeon began to explore for fragments in the pericardial sac. Slipping
her hand under the heart apex, she slid her fingers upward and to the right within the
sac until they were stopped by the cul-de-sac formed by the pericardial reflection near
the base of the heart. Her fingertips were then in the:
A. coronary sinus
B. coronary sulcus
C. costomediastinal recess
D. oblique sinus
E. transverse sinus

29. A hand slipped behind the heart at its apex can be extended upwards until stopped by
a line of pericardial reflection that forms the:
A. Cardiac notch
B. Costomediastinal recess
C. Hilar reflection
D. Oblique pericardial sinus
E. Transverse pericardial sinus

30. The first rib articulates with the sternum in close proximity to the:
A. Nipple
B. Root of the lung
C. Sternal angle
D. Sternoclavicular joint
E. Xiphoid process

31. The portion of the parietal pleura that extends above the first rib is called the :
A. costodiaphragmatic recess
B. costomediastinal recess
C. costocervical recess
D. cupola
E. endothoracic fascia

7
32. You are attending an operation to remove a thymic tumor from the superior
mediastinum. The surgeon asks, "What important nerve lying on and partly curving
posteriorly around the arch of the aorta should we be careful of as we remove this
mass?" You quickly answer, "The--
A. left phrenic
B. left sympathetic trunk
C. left vagus
D. right phrenic
E. right sympathetic trunk

33. A needle inserted into the 9th intercostal space along the midaxillary line would enter
which space?
A. Cardiac notch
B. Costodiaphragmatic recess
C. Costomediastinal recess
D. Cupola
E. Oblique pericardial sinus

34. The pleural cavity near the cardiac notch is known as the:
A. Costodiaphragmatic recess
B. Costomediastinal recess
C. Cupola
D. Hilum
E. Pulmonary ligament

35. The tubercle of the 7th rib articulates with which structure?
A. Body of vertebra T6
B. Body of vertebra T7
C. Body of vertebra T8
D. Transverse process of vertebra T6
E. Transverse process of vertebra T7

36. The ductus arteriosus sometimes remains open after birth requiring surgical closure.
When placing a clamp on the ductus, care must be taken to avoid injury to what
important structure immediately dorsal to it?
A. Accessory hemiazygos vein
B. Left internal thoracic artery
C. Left phrenic nerve
D. Left recurrent laryngeal nerve
E. Thoracic duct

8
37. Blockage of which of the following arteries would lead to ischemia of the apex of the
heart?
A. Anterior interventricular (descending)
B. Left circumflex
C. Posterior interventricular (descending)
D. Right marginal
E. Right coronary

38. A stethoscope placed over the left second intercostal space just lateral to the sternum
would be best positioned to detect sounds associated with which heart valve?
A. aortic
B. pulmonary
C. mitral
D. Tricuspid
E. Inferior vena cava valve

39. Which valves would be open during ventricular systole?


A. Aortic and pulmonary
B. Aortic and tricuspid
C. Mitral and aortic
D. Tricuspid and mitral
E. Tricuspid and pulmonary

40. During fetal life and sometimes persisting into the adult there is an opening between
the right and left atria; this opening is called the:
A. atrioventricular canal
B. ventricular septal foramen
C. foramen ovale
D. sinus venosus
E. truncus arteriosis

41. The heart sound associated with the mitral valve is best heard:
A. In the jugular notch
B. In the second left intercostal space
C. In the second right intercostal space
D. In the fifth left intercostal space
E. To the right of the xiphoid process

42. Which heart valve has leaflets described as "anterior, left and right"?
A. Aortic
B. Pulmonary
C. Left atrioventricular
D. Right atrioventricular
E. Mitral

9
43. Which structure does NOT lie in the coronary sulcus?
A. circumflex artery
B. coronary sinus
C. right coronary artery
D. right marginal artery
E. left coronary artery

44. Which structure contains postganglionic sympathetic fibers?


A. greater splanchnic nerve
B. recurrent laryngeal nerve
C. sympathetic trunk
D. ulnar nerve
E. vagus nerve

45. Which posterior mediastinal structure is most closely applied to the posterior surface
of the pericardial sac?
A. Aorta
B. Azygos vein
C. Esophagus
D. Thoracic duct
E. Trachea

46. In obstruction of the superior or inferior vena cava, venous blood is returned to the
heart by an alternate route via the azygos vein, which becomes dilated in the process.
Which of the following structures might it compress as a result?
A. trachea
B. root of the right lung
C. phrenic nerve
D. thoracic duct
E. descending aorta

47. You are called to perform thoracentesis (remove fluid from the pleural cavity). If you
are to avoid injuring lung or neurovascular elements, where would you insert the
aspiration needle?
A. the top of intercostal space 8 in the midclavicular line
B. the bottom of intercostalspace 8 in the midclavicular line
C. the top of intercostal space 9 in the midaxillary line
D. the bottom of intercostal space 9 in the midaxillary line
E. the top of intercostal space 11 in the scapular line

48. Which feature is found only in the left lung?


A. Cardiac notch
B. Horizontal fissure
C. Oblique fissure
D. Superior lobar bronchus
E. Three lobes

10
49. The oblique fissure of the right lung separates which structures?
A. Lower lobe from lingula
B. Lower lobe from upper lobe only
C. Lower lobe from both upper and middle lobes
D. Lower lobe from middle lobe only
E. Upper from middle lobe

50. A 4-year-old girl is brought in with coughing, and you are told by her mother that she
had been playing with some beads and had apparently aspirated one (gotten it into her
airway). Where would you expect it to most likely be?
A. Apicoposterior segmental bronchus of left lung
B. Left main bronchus
C. Lingular segment of left lung
D. Right main bronchus
E. Terminal bronchiole of right lung, lower lobe

51. Which statement is true about the right lung?


A. it is slightly smaller than the left lung
B. it has a lingular segmental bronchus
C. it occupies the rightmost portion of the mediastinum
D. its upper lobar bronchus lies behind and above the right pulmonary artery
E. it has the right phrenic nerve passing posterior to the lung root

52. You are observing a doctor perform a bronchoscopy. As he passes the bronchoscope
down the trachea, a cartilagenous structure is observed separating the right and left
main stem bronchi. He asks what it is called. You reply that it is called the:
A. Carina
B. Cricoid cartilage
C. Costal cartilage
D. Pulmonary ligament
E. Tracheal ring

53. Which vessel courses across the mediastinum in an almost horizontal fashion?
A. Left subclavian artery
B. Left subclavian vein
C. Left brachiocephalic vein
D. Left internal jugular vein
E. Left common carotid artery

54. Sympathetic fibers in the greater splanchnic nerve arise from neuron cell bodies found
in the:
A. brainstem
B. celiac ganglion
C. chain ganglion
D. spinal cord
E. superior mesenteric ganglion

11
55. Which nerve fiber would have its cell body in the lateral horn of the spinal cord at
segmental level T1?
A. Afferent fiber from cutaneous blood vessels of the nose
B. Afferent fiber from skin around the nipple
C. Efferent fibers to sweat glands in the lumbar region
D. Efferent fibers to skin of the forehead
E. Parasympathetic fibers to the heart

56. Gray rami communicantes contain postganglionic sympathetic fibers that innervate
which of the following structures in the thoracic region?
A. aorta
B. heart
C. lung
D. sweat glands
E. trachea

57. During a procedure to harvest lymph nodes in the posterior mediastinum, the thoracic
duct is accidentally cut. The resulting accumulation of lymph in the pleural cavity is
referred to as:
A. Pleurisy
B. Chylothorax
C. Pyothorax
D. Hemothorax
E. Lymphedema

58. Gaseous exchange occurs in all of the following except:


A alveolus
B tertiary bronchi
C respiratory bronchiole
D alveolar ducts
E alveolar sacs

59. Which of the following are correctly matched?


A elastic cartilage - secondary bronchus
B goblet cell - terminal bronchiole
C smooth muscle - respiratory bronchiole
D dust cell - tertiary bronchus
E ciliated epithelial cell - trachea

60. Which statement regarding lymphocyte circulation is not true:


A both T and B cells are found in peripheral blood
B lymphocytes exit lymph nodes from the convex surface
C T lymphocytes form the periarterial lymphatic sheath (PALS) of the spleen
D lymphocytes enter the thymus via blood vessels
E B lymphocytes are dominant in the lymphoid follicle

12
61. The following lymphoid organs is paired with the appropriate histological feature:
A Spleen - marginal zone
B Pharyngeal tonsil - ciliated pseudostratified columnar epithelium
C thymus - HEV
D Lymph node - Billroth’s cords
E Peyer’s patches -- afferent lymphatic vessels

62. Germinal centres are normally found in the following except:


A spleen
B thymus
C lymph node
D tonsil
E appendix

63. Concerning cilia


A. have an actin core explaining its motility
B. it is attached to the terminal web
C. the base of a cilium is the centriole
D. imotile cilia is seen in the ear
E. it is important in the stomach

64. Which of the statements about the vestibule of the nose is incorrect?
A. it has an elastic cartilage framework
B. it has stratified squamous epithelium
C. it has sweat glands
D. it has cilia on its epithelium
E. it has sebaceous glands

65. The part of the gastrointestinal tract that contains the mucosa-associated lymphoid
tissue (MALT) is the:
A. muscularis mucosae
B. submucosa
C. lamina propria
D. serosa
E. muscularis externa

66. Regarding glands:


A. glands of Littre are sebaceous glands
B. apocrine sweat glands are functional before birth
C. urogastrone is from the duodenal glands
D. intestinal pits are infoldings of the mucosa
E. Brunner’s glands are compound tubuloacinar

13
67. Which is not a feature of monocytes?

A. are the largest of the white cells


B. are precursors of histiocytes
C. it is highly phagocytic in circulating blood
D. are antigen presenting cells
E. have a nucleus with several nucleoli

68. In the alimentary system


A. the center of a classical hepatic lobule is a portal triad
B. In the upper third of the oesophagus the muscular externa consists
of skeletal muscles only
C. The vermiform appendix has Peyers patches
D. Blood flows from the periphery to the center of a portal lobule
E. the gall bladder has submucosal glands

69. Concerning the oral cavity:


A. foliate papillae are numerous on the human tongue
B. odontoblasts degenerate after eruption of the tooth
C. alveolar bone is spongy bone
D. vermillion border has sebaceous glands
E. the hard palate has a loss mucosa

70. Stereocilia
A. are actively moving in waves
B. are part of the respiratory epithelium
C. have a core of microtubules
D. is seen in the uterine tubes
E. have absorptive functions

71. Pilosebaceous unit is made up of the following except:


A. smooth muscles
B. hair follicle
C. sweat gland
D. saccular gland
E. arrector pilorum

72. Which statement is not true concerning eosinophils?


A. have a bilobed nucleus often obscured by bright red granules
B. circulate for about 3-8 hours
C. have a diurnal variation
D. increase in numbers in parasitic disease
E. are phagocytic

73. Regarding the skin


A. it has derivatives of neural crest cells
B. Langers lines are rich with Langerhans cells
C. some sweat glands open in hair follicles
D. sweat glands are seen on the glans penis
E. all sebaceous glands open in hair follicles

14
74. Concerning haemopoiesis:
A. it first begins in the yolk sac
B. it first begins in the liver
C. it first begins in the bone marrow
D. bone marrow has reticulocytes that form reticular fibers
E. megakaryocytes are macrophages

75. The following statement about lymphocytes is incorrect:


A. T lymphocytes have three main subsets
B. CD4 cells are cytotoxic cells killing viual infected cells
C. B lymphocytes are derived from bone marrow
D. Natural killer cells kill viral infected cells
E. there comprise of 20 – 50% of leucocytes in circulation

76. Concerning veins:


A. valves occur in hepatic portal vein
B. they hold about 50% of blood volume at any one time
C. movements of blood towards the heart is aided by contraction of muscles in
their walls
D. veins in the kidneys are fenestrated
E. postcapillary venules are the main sites of diapedesis

77. Which of the following statements concerning the maculae adherentes is incorrect?

A. it is formed by a disc-shaped structure known as a connexon


B. it is important in cell adhesion
C. it is also known as desmosomes
D. it is also called a spot junction
E. it is associated with intermediate filaments

78. Stratified columnar epithelium is found in the following:


A. oesophagus
B. gall bladder
C. epididymis
D. conjunctiva
E. ducts

79. The organelle/s that divide by fission is or are the


A. ribosome
B. microbody
C. mitochondrion
D. Golgi apparatus
E. smooth endoplasmic reticulum

15
80. Which statement concerning erythrocytes is not true?
A. their nuclei are not extruded prior to release into circulation
B. mature erythrocytes do not have any cytoplasmic organelles
C. they have a pale-staining central region
D. effete cells are removed from the circulation by the spleen and liver
E. synthesis of haemoglobin is completed before the cells are released into
the circulation

81. Regarding reticulocytes:


A. these are connective tissue cells of the bone marrow
B. they are easily distinguished from mature erythrocytes in routinely stained
blood smears
C. their count provides a measure of the rate of red blood cell formation in the
bone marrow
D. form the framework in the thymus
E. their number decreases following bleeding

82. The following concerning neutrophils are true except:


A. constitute 40-75% of circulating leucocytes
B. in peripheral blood films have an appendage on one of the nuclear lobes in
3% of females
C. have secondary granules which are large lysosomes
D. are also called pus cells
E. have a degree of protein synthesis

83. The nervous system:


A has CNS – made up of the brain only
B the peripheral nervous system (PNS) is a completely separate
anatomical entity from the CNS
C the autonomic nervous system is part of the PNS
D cell bodies in the CNS are seen in grey matter
E cell bodies of neurons outside the CNS form ganglia

84. Mucous cells:


A. are usually cuboidal in shape
B. have nuclei that are pressed towards the bases of the cells
C. almost exclusively comprise the secretory portion of the submandibular gland
D. are basophilic
E. are seen in breast tissue

85. The following skin appendages are seen in thick skin:


A sweat glands
B hair follicles
C sebaceous glands
D apocrine sweat glands
E smooth muscle

16
86. The following cell junction complexes are continuous except:
A. zonula adherens
B. zonula occludens
C. intermediate junction
D. macula adherens
E. tight junction

87. The terminal ends of the ilioinguinal nerves in the female are referred to as:
A. Anterior cutaneous branches
B. Anterior labial
C. Cremasterics
D. Iliohypogastrics
E. Pampiniform plexus

88. The usual location for an appendectomy incision is the:


A. umbilical
B. right lateral
C. right lower quadrant
D. right upper quadrant
E. right hypochondrium

89. The inferior border of the rectus sheath posteriorly is called the:
A. Falx inguinalis
B. Inguinal ligament
C. Internal inguinal ring
D. Arcuate line
E. Linea alba

90. A medical student was asked by her preceptor to palpate the margin of the superficial
inguinal ring of a healthy male patient. After passing her finger down the edge of the
medial crus of the superficial inguinal ring, she felt a bony protuberance deep to the
lateral edge of the spermatic cord, which she correctly identified as the :
A. pecten pubis
B. pubic symphysis
C. pubic tubercle
D. iliopubic eminence
E. iliopectineal line

91. Which structure passes through the deep inguinal ring?


A. Iliohypogastric nerve
B. Ilioinguinal nerve
C. Inferior epigastric artery
D. Medial umbilical ligament
E. Round ligament of the uterus

17
92. A loop of bowel protrudes through the abdominal wall to form a direct inguinal
hernia; viewed from the abdominal side, the hernial sac would be found in which
region?
A. Deep inguinal ring
B. Lateral inguinal fossa
C. Medial inguinal fossa
D. Superficial inguinal ring
E. Supravesical fossa

93. The superficial inguinal ring is an opening in which structure?


A. External abdominal oblique aponeurosis
B. Falx inguinalis
C. Internal abdominal oblique muscle
D. Scarpa's fascia
E. Transversalis fascia

94. During exploratory abdominal surgery on a 55-year-old male complaining of right


lower quadrant pain, the surgeon initially sees no appendix but knows that he can
quickly locate it by
A. looking at the confluence of the teniae coli
B. palpating the ileocecal valve and looking just above it
C. palpating lymph nodules
D. removing the right layer of the mesentery of the jejunoileum
E. palpating and inspecting the pelvic brim

95. Meckel's diverticulum:


A. is an abnormal persistance of the urachus
B. is a site of ectopic pancreatic tissue
C. is caused by a failure of the midgut loop to return to the abdominal cavity
D. is an abnormal connection of the midgut to the duodenum
E. is associated with polyhydramnios

96. The following organs are derived from mesoderm except:


A. skeletal musculature
B. musculature of blood vessels
C. cardiac musculature
D. musculature of the stomach
E. suprarenal medulla.

97. In the foot, the term adduction implies movement of toes towards
the………………………..
A. First toe
B. Second toe
C. Third toe
D. Fifth toe
E. Fourth

98. Which of the planes specifically divides the body into unequal right and left parts?

18
A. Sagittal
B. Parasagittal
C. Mid-sagittal
D. Coronal
E. Lambdoid

99. What does the positional term proximal mean?


A. Near the surface of the body
B. Towards the body trunk
C. Close to the center
D. Away from the midline
E. Towards the tail

100. Which of the following anatomical terms means ‘on different sides of the
body’?
A. Sagittal
B. Ipsilateral
C. Contralateral
D. Lateral
E. Deep

101. Identify the correct order of tissue preparation steps.


A. Fixation, Dehydration, Clearing, Infiltration, Embedding and Trimming
B. Fixation, Clearing, Dehydration, Infiltration, Embedding and Trimming
C. Clearing, Fixation, Dehydration, Infiltration, Embedding and Trimming
D. Clearing, Fixation, Dehydration, Embedding, Infiltration and Trimming
E. Clearing, Dehydration, Fixation, Infiltration, Embedding and Trimming

102. Which of the following cell organelles are non-membrane bound?


A. Vesicle
B. Peroxisomes
C. Proteasomes
D. Lysosomes
E. Nucleus

103. Which of the following is NOT true concerning mitochondria?


A. They replicate independently from the cell
B. They are passed on via the sperm
C. Contain their own genetic material
D. Vary in morphology between cells
E. It makes proteins

19
104. Which of the following has a membrane which is continuous with that of the
nucleus?
A. Smooth endoplasmic reticulum
B. Golgi apparatus
C. Rough endoplasmic reticulum
D. Heterosome
E. Centrosome

105. Which of the following are abundant in fibroblasts?


A. Free ribosomes
B. Proteasomes
C. Rough endoplasmic reticulum
D. Golgi apparatus
E. Proteasomes

106. Which of the following statements is correct?


A. DNA material that is inactive in RNA synthesis is referred to as euchromatin
B. The nucleus is structurally supported by microtubules called nuclear lamina
C. Chromatin is made up of heterochromatin and histone proteins
D. All the nucleoproteins are synthesised in the cytoplasm
E. None of the above

107. ……………………………… are numerous in highly metabolic cells.


A. Ribosomes
B. Smooth endoplasmic reticulum
C. Proteasomes
D. Rough endoplasmic reticulum
E. Mitochondria

108. Concerning the cell cycle


A. In G1 phase the cells are functioning normally
B. M phase cells are at rest
C. G0 phase cells synthesize DNA
D. A neuron is an example of a cell at G2 phase
E. S phase is characterised by cell division

109. Which of the following is true about glands?


A. The glands in the large intestines are simple acinar glands
B. The mammary glands are compound tubular glands

20
C. Brunner’s glands are compound acinar
D. Stomach glands are simple branched tubular
E. The parotid gland is a mixed gland

110. The term opposition means moving the ……………………….


A. Fourth and fifth fingers towards the middle finger
B. Little finger towards the index finger
C. Medial and lateral fingers away from the middle finger
D. Thumb towards the other fingers
E. Long finger towards the little finger

21
SECTION II: DIAGRAMS 54

101. Below is a diagram of a certain tissue label the parts A to L in the spaces provided

A------------------------------------------------- B------------------------------------------------------

C------------------------------------------------- D-----------------------------------------------------

E------------------------------------------------- F------------------------------------------------------

G------------------------------------------------- H------------------------------------------------------

I------------------------------------------------- J------------------------------------------------------

K------------------------------------------------- L------------------------------------------------------

22
102. Below is a diagram of a histological section answer the questions

Name the parts

A------------------------------------------------- B-------------------------------------------------

C------------------------------------------------- D-------------------------------------------------

E. Name the tissue and structure shown----------------------------------------------------------

F. Name the structures found in A----------------------------------------------------------------

23
103. Below is the diagram of the heart showing its anterior view. Label the structures shown
A to F.

A------------------------------------------------- B------------------------------------------------------

C------------------------------------------------- D-----------------------------------------------------

E------------------------------------------------- F------------------------------------------------------

24
104. Below is a diagram of the brachial . Label the parts A to D in the spaces provided

A------------------------------------------------- B------------------------------------------------------

C------------------------------------------------- D-----------------------------------------------------

25
26
1 arcade artries

2.Middle colic

3.Superior mesenteric

27
END OF EXAMINATION

28
THE UNIVERSITY OF ZAMBIA

UNIVERSIT EXAMINATIONS –05/10/ 2018

HUMAN ANATOMY

HAN 3010

TIME ALLOWED: 3 HOURS MAXIMUM MARKS: 195

INSTRUCTIONS TO STUDENTS

1. Attempt ALL questions


2. Read the instructions to each Section.
3. Write your computer number on each paper submitted

COMPUTER NUMBER:_________________________

1
SECTION A 100 Marks

Select the one most appropriate answer from the alternatives A-D in each
question, by marking X on the answer sheet grid provided. There is no penalty for
wrong answers.

1. What does the term inversion mean?


A. Dorsi-flexion of the foot
B. Turning of the foot to make the sole face medially
C. Plantar-flexion of the foot
D. Turning of foot to make its sole face laterally

2. Which of the following terms best describes the movement of the great toe
towards the rest of the toes?
A. Protraction
B. Pronation
C. Adduction
D. Retraction

3. What does the term ‘parfocal’ mean?


A. The resolving power of an objective lens
B. Focusing your specimen using high magnification
C. Changing the objectives without refocusing
D. Using the coarse adjusting knob after moving from x10 to x40 objective

4. Which of the following is correct regarding the terms ‘resolving power’?


A. The ocular lens determines the resolving power of a microscope
B. It is the largest distance between two particles at which they appear as
separate objects
C. It is the ability of the microscope to magnify many objects at the same time
D. The objective with the high magnification has a high resolving power

5. Which of the following statements rightly describes a plane?


A. It is a solid line that is drawn on the body of a subject
B. It is the cut surface of the body.
C. The coronal and the midsagittal sections are examples
D. An imaginary flat surface dividing the body

6. Which of the following organelles are abundant in fibroblasts?


A. Free ribosomes
B. Proteasomes
C. Rough endoplasmic reticulum
D. Golgi apparatus

2
7. ……………………………… are numerous in cells that are involved in
detoxification e.g. hepatocytes.
A. Ribosomes
B. Smooth endoplasmic reticulum
C. Proteasomes
D. Mitochondria

8. The fluid mosaic model describes the plasma membrane as consisting of


A. Two layers of phospholipids with protein in between them
B. A protein bilayer with embedded phospholipids
C. A phospholipid single layer with embedded proteins
D. A phospholipid bilayer with proteins embedded in it

9. Which statement is correct about the cell cycle?


A. In G1 phase the cells are resting
B. In M phase, organelles mature
C. In G0 phase DNA is duplicate
D. In G2 karyokinesis occurs

10. Regarding the cell, which of the following is not true?


A. Neutrophils contain zymogenic granules
B. Macrophages have numerous lysosomes
C. Skeletal muscle cells have a prominent nucleolus
D. Basophils compliment mast cell in terms of function

11. Which of the following is correct concerning myoepithelial cells?


A. Are stimulated by hormones
B. Are smooth muscle cells
C. Surround the basal lamina
D. Are scattered among secretory cells

12. …………………………… line is seen as a line of demarcation between lighter


flexor and the darker extensor parts of the skin of limbs.
A. Futcher’s
B. Wrinkle
C. Occupational
D. Contour

13. Regarding the resting skin tension lines, which statement is correct?
A. Are inappropriate for surgical incision
B. Pinching at right angle will produce more tension lines
C. Are altered by contraction of muscles deep to them
D. Are same as kraissl lines

3
14. Skin creases limited to the cornified (uppermost layer of the epidermis) layer are
termed…………………………..
A. Primary
B. Secondary
C. Tertiary
D. Quaternary

15. Brunner’s glands are an example of …………………………………….


A. Simple acinar
B. Simple branched acinar
C. Compound branched tubular
D. Compound tubulo-acinar

16. Which of the following structures give the nucleus its structural support
framework?
A. Laminins
B. Microfilaments
C. Actins
D. Keratins

17. Which one of the following statements regarding cytoskeletons is correct?


A. The intermediate filaments are the smallest form of cytoskeletons
B. Microfilaments anchor in position some plasma membrane proteins
C. A pair of centrosomes make a centriole
D. Basal bodies in a cilium are made of nine pairs of microtubules

18. Which of the following epithelium lines the cornea of the eye?
A. Simple squamous epithelium
B. Pseudostratified columnar ciliated epithelium
C. Stratified squamous non-keratinized epithelium
D. Stratified columnar epithelium

19. Which of the following epithelium lines the pleural cavity?


A. Simple squamous epithelium
B. Pseudostratified columnar ciliated epithelium
C. Endothelium
D. Simple columnar epithelium

4
20. Which of the following glands release their secretory products by the apocrine
mode of secretion?
A. Mammary
B. Sebaceous
C. Eccrine sweat glands
D. Pituitary

21. With regard to the lining of the ileum, the following are true except………..
A. It is lined by a simple columnar epithelium
B. It has multicellular glandular structures, goblet cells
C. It has simple straight tubular glands, crypts of lieberkuhn
D. Paneth cells are present at the base of the intestinal glands

22. Which of the following epithelia withstands desiccation?


A. Stratified squamous non-keratinised
B. Pseudostratified columnar with stereocilia
C. Stratified squamous keratinised
D. Pseudostratified columnar ciliated

23. Which of the following is a follicular endocrine gland?


A. Pituitary
B. Thyroid
C. Pancreas
D. Adrenal

24. Which of the following cells have receptors for immunoglobulin E on their
surface?
A. Plasma
B. T-lymphocytes
C. Basophils
D. B-lymphocytes

25. Concerning the perichondrium, which of the following statements is correct?


A. It consists of collagen type II fibres
B. It has an inner cell layer consisting of chondrocytes
C. It makes the outer covering of fibrocartilage
D. It has progenitor cells which become chondroblasts

26. Cartilage found in the external acoustic meatus is characterised by……


A. Cell nests of not more than two chondrocytes
B. Absence of perichondrium
C. Collagen type II fibres only
D. The same type of cartilage as that in the pubic symphysis

5
27. With cartilage regard to epiphyseal, which of the following is true?
A. It is fibrocartilage
B. Its arrest has no effect on growth of long bones
C. It is radiolucent
D. Its resting stage is characterised by osteocytes

28. Which of the following statements is correct about cartilage?


A. The territorial matrix is not the matrix that immediately surrounds the lacuna
B. The interteritorrial matrix has no staining differences with the territorial
matrix
C. The territorial matrix is rich in glycosaminoglycans (GAGs)
D. Collagen fibres are less abundant in the interterritorial matrix

29. Compact bone is characterised by one of the following;


A. 9Osteons, also known as Harvesian canals
B. A cement line surrounding each Harvesian system
C. Osteocytes actively secreting extracellular matrix
D. Interconnected trabeculae of bone

30. One of the following is not true concerning woven bone.


A. It is the first type of bone to appear in embryonic life
B. It is replaced by lamellar bone in adult life
C. In adults it is found near sutures of the skull
D. It has same mineral content as lamellar bone

31. In intramembranous ossification,


A. Mesenchymal cells differentiate into osteoprogenitor cells
B. There is replacement of hyaline cartilage by bone tissue
C. In cranial flat bones, the bone in the centre forms faster than the periphery
D. Osteocytes lose their contact with each other once bone has developed

32. Which of the following is correct regarding the trapezius muscle?


A. It is a flat diamond shaped muscle
B. It is innervated by the dorsal scapular nerve
C. It sends its proprioceptive fibres via C3 and C4
D. It makes up the superficial group of muscles of the back

33. To which of the following muscles of the back is the thoracodorsal nerve
immediately deep?
A. Latissmus
B. Trapezius
C. Rhomboid major
D. Rhomboid minor

6
34. Loss of patella reflex and cutaneous sensation on the anteromedial side of leg
indicate damage to spinal nerve:
A. L4
B. L5
C. S2
D. S3

35. Which of the following muscles is not a medial rotator of the leg?
A. Semitendinosus
B. Semimebranosus
C. Biceps femoris
D. Gracilis

36. Mulenga sustained an injury in the popliteal fossa injuring the tibial nerve. This
might result in:
A. Loss of eversion
B. Loss of sensation between
C. Inability to stand on one’s toes
D. Foot drop

37. Which of the following is correct about the fibular artery:


A. Supplies the muscles of the anterior compartment of the leg
B. Passes anterior to the interosseous membrane
C. Usually becomes the dorsalis pedis
D. Courses through the deep posterior compartment of the leg

38. A tight plaster cast that exerted pressure on the head and neck of the fibula might
result in loss of:
A. Eversion
B. Plantar-flexion
C. Inversion
D. Flexion of the hallux

39. Where would you feel for the pulse of the dorsalis pedis artery?
A. Directly superficial to the intermediate cuneiform
B. Directly lateral to the tendon of the extensor digitorum longus
C. Directly in between the talus and the navicular bones
D. Directly medial to the extensor halluces longus

40. The blood supply to the posterior compartment muscles of the thigh is by:
A. Femoral artery
B. Popliteal artery
C. Profunda femoris artery
D. Obturator artery

7
41. The spinal cord segment that supplies cuteneous innervation to the blateral side of
the foot is:
A. L4
B. L5
C. S1
D. S2

42. The muscle that contracts to unlock the extended knee joint is the:
A. Popliteus
B. Plantaris
C. Medial head of gastrocnemius
D. Lateral head of gastrocnemius

43. Which of the following correctly describes the pectoralis major muscle?
A. It is made up of two parts, the clavicular and the sternocostal parts.
B. It is superficially covered by the clavipectoral fascia
C. It divides the axillary artery into the first, second and third parts
D. It is innervated by the lateral and medial pectoral nerves.

44. With regard to the vagi nerves, which statement is True?

Left vagus is anterior to the phrenic nerve


A. Left recurrent laryngeal hook around the left subclavian artery
B. Right recurrent laryngeal, hook around the ligamentum
arteriosum
C. Left vagus passes posterior to the root of the left lung.

45. Where does the right sympathetic chain cross the diaphragm as it enters the
abdomen?
A. Through the aortic hiatus
B. Posterior to the diaphragm
C. Through the inferior vena caval opening
D. Through the oesophageal opening

46. Which of the following is correct concerning coronary vessels?


A. The right coronary artery always supplies the sinuatrio node
B. The coronary sinus is the only vessel that drains the heart walls
C. The coronary veins drain into the inferior vena cava
D. The left coronary artery partly supplies the right ventricle

47. Which of the following is NOT a characteristic of the pericardium?


A. It has a serous part
B. Pericarditis may cause accumulation of fluids in the pericardial cavity
C. The visceral pericardium is the inner most part

8
D. The fibrous pericardium is supplied by the vagus nerve

48. Which of the following organs is likely to be compressed in a patient with an


abnormally dilated left atrium?
A. Esophagus
B. Trachea
C. Thoracic duct
D. Azygos vein

49. In the posterior triangle of the neck,


A. The scalenusmedius is superior to the levator scapulae
B. The trunks of the brachial plexus are in between the scalenus anterior and
medius muscles
C. The semi-spinaliscapitis is inferior to the splenius capitis
D. The accessory nerve emerges below the midpoint of the posterior border of
the sternocleidoastoid muscle.

50. Mr Munalula comes to the hospital with a “swelling” on the left upper lateral part
of his back. On examination, you find that it is a ‘winged’ left scapula. Which of
the following nerves is likely to have been affected?
A. Dorsal scapular
B. Accessory
C. Thoracodorsal
D. Long thoracic

51. You have a patient with a stab wound immediately above the medial third of the
left clavicle. Which of the following are you worried of?
A. Injury to the trachea
B. Pneumothorax
C. Perforation of the superior vena cava
D. Injury of the trunks of the brachial plexus

52. Which of the following is not a normal constriction of the esophagus?


A. Diaphragmatic
B. Bronchial
C. Tracheal
D. Pharyngo-esophageal

9
53. What is the order of testicular coverings from superficial going deep.
A. Skin, dartos muscle, cremateric fascia, external spermatic fascia then
internal spermatic fascia
B. Skin, cremateric fascia, dartos muscle, external spermatic fascia then
internal spermatic fascia
C. Skin, dartos muscle, internal spermatic fascia cremateric fascia, external
spermatic fascia then
D. Skin, dartos muscle, external spermatic fascia, cremateric fascia then
internal spermatic fascia

54. Which of the following structures do you expect to find on the skin of a premature
baby?
A. Vellus
B. Terminal hair
C. Lanugo
D. Coarse hair

55. Concerning skin glands,


A. A seven months old fetus has functional Sebaceous glands
B. In women, eccrine sweat glands are affected by hormones
C. Apocrine gland are active from birth
D. Eccrine sweat glands are straight tubular glands

56. Moses twists Maria’s forearm as they were playing. Which of the following
sensory structures of her skin were able to sense the twist?
A. Pacinian corpuscles
B. Krause end bulbs
C. Ruffini corpuscles
D. Meissner corpuscles

57. Which statement is FALSE regarding the heart?


A. The apex of the heart points anteriorly, inferiorly to the left
B. The apex of the heart is completed formed by the left ventricle
C. The base of the heart lies on the inferior aspect of the heart
D. The heart is obliquely oriented with reference to the median plane

58. Which of the following rightly describes the heart beat?


A. Ventricular systole proceeds atrial systole
B. The sinuatrial node depends on the brain to generate impulses
C. The apex of the heart is the last one to contract in one beat
D. Rise in ventricular blood volume closes the mitral valve

10
59. Regarding the lymphatic vessels,
A. The cisterns chyli lies anterior to the body of L1 on the right side and
posterior the abdominal aorta
B. Thoracic duct enters the thorax through the esophageal hiatus
C. The thoracic duct terminates at the union of the right and left brachiocephalic
veins
D. The cisterna chyli has colorless lymph

60. Which of the following is not one of the main vessels that drain into the cisterna
chyli?
A. Left lumbar trunk
B. Right lumbar trunk
C. Posterior intercostal trunk
D. Intestinal trunk

61. The mediastinal surface of each lung has a cardiac impression. Which of the
following statements correctly describes this impression?
A. On the right lung, the right ventricle forms the shallow impression
B. On the right lung, the deeper impression is produced by the right atrium
C. On the left lung, a deeper impression is produced by the left atrium
D. On the left lung, the left ventricle produces the deeper impression

62. Which of the following is correct concerning cardiac innervation?


A. The superficial cardiac plexus is formed by fibres from all the cervical
sympathetic ganglia
B. The superficial cardiac plexus is formed by fibres from all the cervical
sympathetic ganglia and the recurrent laryngeal nerve
C. The superficial cardiac plexus is formed by fibres from the left superior
cervical sympathetic ganglion and the inferior cardiac branch of the left
vagus
D. The superficial cardiac plexus is formed by fibres from the right superior
cervical sympathetic ganglion and the inferior cardiac branch of the right
vagus

63. Which of the following is not true regarding the kidneys?


A. They are retroperitoneal
B. The renal sinuses separate the renal pyramids
C. The Gerota’s fascia surrounds the fatty capsule
D. The renal corpuscles are found in the cortex

11
64. For a molecule to move from the blood stream to the Bowman’s space to become
the glomerular filtrate, it passes through the following structures except?
A. Podocytes
B. Filtration slit diaphragms
C. Endothelium
D. Fused basal laminae of capillaries and visceral layer of Bowman’s capsule

65. Concerning functions of the Juxtaglomerular Apparatus (JGA) all of the following
are correct EXCEPT
A. It produces rennin
B. It produces angiotensin
C. Involved in autoregulation of glomerular filtration rate
D. Located in the distal proximal convoluted tubule

66. Which of the following structures are not a characteristic of the visceral part of the
Bowman’s capsule?
A. Podocytes
B. Primary processes
C. Pedicels
D. Vascular pole

67. Which of the following cells in the kidneys are involved in the process of red blood
cell production?
A. Mesangial cells
B. Juxtaglomerular granular cells
C. Extraglomerular mesangial cells
D. Fibroblasts

68. Which of these is not a function of the kidneys?


A. Regulation of acid-base balance
B. Secretion of angiotensin I
C. Activation of Vitamin D
D. Balance of water and electrolytes

69. Which of the following structures are not found in the renal corpuscle?
A. Podocytes
B. Capillaries
C. Macula densa
D. Mesangial cells

70. Concerning the structure of the male reproductive system,


A. The urethra passes through the corpus cavernosa
B. The penis has paired corporaspongiosa
C. The glans extends from the corpus spongiosum

12
D. Erectile tissue of the penis is made up of skeletal muscles
71. Which of the following structures have a lot of stereocilia on their epithelium?
A. Epididymis
B. Ductus deferens
C. Prostatic urethra
D. Ejaculatory duct

72. Which of the following is incorrect regarding Sertoli cells?


A. They extend from the basal lamina to the luminal surface
B. The spermatids are invaginated into the Sertoli cells
C. They produce follicle stimulating hormone (FSH)
D. They phagocytize excess cytoplasm from the spermatids

73. Which statement is true concerning seminal vesicles?


A. They produce 30% of the ejaculate
B. The fructose in its secretion coagulates the semen after ejuclation
C. The prostaglandins in its secretion stimulate activity in the female
D. They store spermatozoa

74. What is the name of the capsule that surrounds the testes
A. tunica albuginea
B. tunica adventitia
C. tunica media
D. tunica vaginalis

75. Maturation of spermatozoa occurs in the


A. Seminiferous tubules
B. Epididymis
C. Rete testis
D. Vas deferens

76. Which of the following cells are not found in the interstitial tissue of the testes
A. Fibroblasts
B. Sertoli
C. Myoid
D. Leydig

77. Which of the following cells of the testes produce the hormone testosterone?
A. Leydig cells
B. Sertoli cells
C. Myoid cells
D. Mast cells

13
78. During penile erection, which nervous system causes vasodilation of blood vessels
of the penis?
A. Parasympathetic
B. Sympathetic
C. Somatic
D. Both sympathetic and parasympathetic

79. Which of the following is correct?


A. Luteinizing hormone stimulates Sertoli cells to release testosterone
B. Follicle stimulating hormone (FSH) stimulate Leydig cells of the interstitium
C. Testosterone stimulate Leydig cells in the seminiferous tubules
D. Inhibin produced by Sertoli sells causes reduction in FSH

80. Which of the following organs is considered as a primary sex organ in males?
A. Penis
B. Testis
C. Scrotum
D. Prostate gland

81. Concerning seminiferous tubules of the testes


A. Converge to become epididymis
B. Lined by simple cuboidal epithelium
C. Basement membrane covered by skeletal muscle fibres
D. They produce spermatozoa

82. Concerning the Vas deferens


A. Enlarged portion is called ampulla
B. Lined by simple columnar epithelium with stereocilia
C. Sperms are stored in the distal portion
D. Enters abdominal cavity via pubic arch

83. Which of the following hormones is responsible for ovulation


A. Progesterone
B. Luteinizing hormone
C. Follicle stimulating hormone
D. Estrogen

84. What gland in males is homologous to greater vestibular gland in females


A. Prostate gland
B. Bartholin’s gland
C. Cowper’s gland
D. Urethral glands

14
85. Female reproductive system
A. The epithelial cells of the uterus have microvilli
B. Uterine tubes epithelial cells are ciliated
C. The uterine glands are simple tubular
D. During menstruation, pars functionalis of the endometrium is not lost

86. In which of the following structures does fertilization occur?


A. Fimbriae
B. Infundibulum
C. Ampulla
D. Isthmus

87. The following joint is a syndesmosis:


A. Distal tibiofibular
B. Proximal tibiofibular
C. Interphalangeal
D. Humeroulnar

88. The sternal angle of Louis is an important anatomical landmark on the sternum that
can be used to count ribs. What is its vertebral level?
A. C3/C4
B. C5/C6
C. T2/T3
D. T4/T5

89. Which of the following statements is correct about the nervi erigentes?
A. Consists of somatic fibres from sacral spinal nerves S2, S3, S4
B. Consists of postganglionic fibres from sacral spinal nerves S2, S3, S4
C. Responsible for motor supply of the detrusor muscle
D. Supplies the external urethral sphincter

90. Which of the following is correct about the gray rami communicantes?
A. Consists of preganglionic parasympathetic fibres
B. Nerve fibres are largely unmyelinated
C. Carries preganglionic sympathetic fibres
D. Fibres arise from the anterior horn cells of the spinal cord

91. Which vertebral level is safe to perform a lumbar puncture in a neonate?


A. L3/L4
B. L1/L2
C. L4/L5
D. L5/S1

15
92. Which of the following nerves is likely to be injured in surgical fracture of the
humerus?
A. Radial nerve
B. Axillary nerve
C. Median nerve
D. Musculocutaneous nerve

93. At which joint does pronation and supination of the forearm take place?
A. Humero-ulnar joint
B. Humeroradial joint
C. Proximal radioulnar joint
D. Radiocarpal joint

94. Which nerve runs in close proximity to the medial epicondyle of the humerus?
A. Radial nerve
B. Median nerve
C. Musculocutaneous nerve
D. Ulnar nerve

95. Which of the following statements concerning the corpus luteum is true?
A. It produces LH.
B. It produces FSH.
C. It derives its granulosa luteal cells from the theca externa.
D. It becomes the corpus albicans.

96. LH exerts which one of the following physiological effects?


A. It triggers completion of the second meiotic division by secondary oocytes.
B. It triggers ovulation.
C. It suppresses release of estrogens.
D. It induces primary follicles to become secondary follicles.

97. The basal layer of the uterine endometrium


A. Is sloughed during menstruation.
B. Has no glands.
C. Is supplied by coiled arteries.
D. Is supplied by straight arteries.

98. Which of the following statements concerning adrenal parenchymal cells is true?
A. Those of the zona fasciculata produce androgens.
B. Those of the adrenal medulla produce epinephrine and norepinephrine.
C. Those of the zona glomerulosa produce glucocorticoids.
D. Those of the cortex contain numerous secretory granules.

16
99. Characteristics of pinealocytes include which one of the following?
A. They produce melatonin.
B. They resemble astrocytes.
C. They contain calcified concretions of unknown function.
D. They act as postganglionic sympathetic cells.

100. Prolactin is synthesized and secreted by which of the following cells?


A. Acidophils in the pars distalis
B. Basophils in the pars tuberalis
C. Somatotrophs in the pars distalis
D. Basophils in the pars intermedia

17
SECTION B 20 Marks

Select the one most appropriate answer from the alternatives A-E in each
question, by marking X on the answer sheet grid provided. There is no penalty for
wrong answers.

1. Which of the following is a major characteristic of meiosis I?


A. Splitting of the centromere
B. Pairing of homologous chromosomes
C. Reducing the amount of DNA to 1N
D. Achieving the diploid number of chromosomes
E. Producing primordial germ cells

2. In the production of male gametes, which of the following cells remains dormant
for 12 years?
A. Primordial germ cell
B. Primary spermatocyte
C. Secondary spermatocyte
D. Spermatid
E. Sperm

3. All primary oocytes are formed by


A. week 4 of embryonic life
B. month 5 of fetal life
C. birth
D. month 5 of infancy
E. puberty

4. When does formation of primary spermatocytes begin?


A. During week 4 of embryonic life
B. During month 5 of fetal life
C. At birth
D. During month 5 of infancy
E. At puberty

5. Concerning maturation of the female gamete (oogenesis), when do the oogonia


enter meiosis I and undergo DNA replication to form primary oocytes?
A. During fetal life
B. At birth
C. At puberty
D. With each ovarian cycle
E. Following fertilization

18
6. Where do primordial germ cells initially develop?
A. In the gonads at week 4 of embryonic development
B. In the yolk sac at week 4 of embryonic development
C. In the paraxial mesoderm at month 4 of embryonic development
D. In the intermediate mesoderm at month 4 of embryonic development
E. In the gonads at puberty

7. Which of the following structures degenerate during ‘hatching’?


A. Endometrium in progestational phase
B. Zona pellucida
C. Syncytiotrophoblast
D. Cytotrophoblast
E. Functional layer of the endometrium

8. Extraembryonic mesoderm is found between which two layers?


A. Epiblast and hypoblast
B. Syncytiotrophoblast and cytotrophoblast
C. Syncytiotrophoblast and endometrium
D. Exocoelomic membrane and syncytiotrophoblast
E. Exocoelomic membrane and cytotrophoblast

9. How much DNA does a primary spermatocyte contain?


A. 1N
B. 2N
C. 4N
D. 6N
E. 8N

10. When after fertilization does implantation begin?


A. morula stage
B. By 12 hours
C. By day 1
D. By day 2
E. By day 7

11. Which of the following are components of the definitive chorion?


A. Extraembryonic somatic mesoderm and epiblast
B. Extraembryonic somatic mesoderm and cytotrophoblast
C. Extraembryonic somatic mesoderm and syncytiotrophoblast
D. Extraembryonic somatic mesoderm, cytotrophoblast, and syncytiotrophoblast
E. Extraembryonic visceral mesoderm, cytotrophoblast,and syncytiotrophoblast

12. The lateral mesoderm is divided into two distinct layers by the formation of the
A. chorionic cavity
B. intraembryonic coelom
C. amniotic sac
D. notochord
E. yolk sac

19
13. Somites may differentiate into which of the following?
A. Urogenital ridge
B. Kidneys
C. Notochord
D. Epimeric muscles
E. Epithelial lining of the gastrointestinal tract

14. Intermediate mesoderm will give rise to the


A. neural tube
B. heart
C. kidneys and gonads
D. somites
E. vertebral column

15. Concerning spermatogenesis


A. at birth the seminiferous tubules don’t have a lumen
B. Sertoli cells are the only cells present at birth
C. Primary Spermatocytes have a short prophase I
D. Spermatids divide to form mature spermatozoa
E. Spermatogonia migrate from posterior abdomen to testis at puberty

16. Concerning neuralation


A. the neural plate is formed during the second week
B. the primitive streak is the important inductor
C. the neural crest cells are important in the formation the adrenal medulla
D. the cranial neural pore closes at 28 day
E. Folic acid is not important

17. Which of the following is not true concerning oocyte transport


A. the fimbriated end of the uterine tube sweeps over the ovary at ovulation
B. the uterine tube undergoes peristalsis
C. the ovulated oocyte has amoebic movement
D. cilia helps in transport
E. the peg cells of the uterine tube play a role

18. Serous membranes


A. develop from the chorionic cavity
B. are lateral plate mesoderm derivatives
C. are ectodermal in origin
D. dura mater is one example
E. the peritoneum is derived from the yolk sac

19. Which of the following structures is not present at term


A. decidua basalis
B. decidua capsularis
C. decidua parietalis
D. Chorion
E. amniotic membrane

20
20. Which is not true concerning the Persistent truncus arteriosus
A. results when the conotruncal ridges fail to fuse and to descend toward the
ventricle
B. the pulmonary artery arises some distance above the origin of the undivided
truncus.
C. the persistent truncus is always accompanied by a defective interventricular
septum.
D. The undivided truncus overrides both ventricles and receives blood from both
sides.
E. is always associated with an atrial septal defect

SECTION C 30 Marks
SHORT ANSWER SECTION
1. Give any four characteristics of products of meiosis
………………………………………………………………………………………
………………………………………………………………………………………
………………………………………………………………………………………
………………………………………………………………………………………
………………………………………………………………………………………
………………………………………

2. Define: Macula pelucida/stigma


………………………………………………………………………………………
………………………………………………………………………………………
………………………………………………………………………………………
………………………

3. Define: Corpus luteum graviditatis


………………………………………………………………………………………
………………………………………………………………………………………
………………………………………………………………………………………
………………………

4. List the contents of the femoral sheath


………………………………………………………………………………………
………………………………………………………………………………………
………………………………………………………………………………………
………………………………………………………………………………………
………………………………………………………………………………………
………………………………………………………………………………………
………………………………………………

21
5. List 5 functions of the placenta
………………………………………………………………………………………
………………………………………………………………………………………
………………………………………………………………………………………
………………………………………………………………………………………
………………………………………………………………………………………
………………………………………………………………………………………
………………………………………………………………………………………
………………………………………………………

6. Name the 5 structures that pass through the primitive umbilical ring
(amnio-ectodermal) junction at the fifth week of development
………………………………………………………………………………………
………………………………………………………………………………………
………………………………………………………………………………………
………………………………………………………………………………………
………………………………………………………………………………………
………………………………………………………………………………………
………………………………………………………………………………………
………………………………………………………

7. Name the derivatives of the following


i. First aortic arch artery
…………………………………………………………………………………
…………..............................................................................................................
......................
ii. Third aortic arch artery
…………………………………………………………………………………
…………………………………………………………………………………
………………
iii. Fourth aortic arch artery (right)
…………………………………………………………………………………
…………………………………………………………………………………
………………
iv. Distal part of the left sixth aortic arch artery
…………………………………………………………………………………
…………………………………………………………………………………
………………
v. Left horn of the sinus venosus
…………………………………………………………………………………
…………………………………………………………………………………
………………
vi. The right common cardinal vein
…………………………………………………………………………………
…………………………………………………………………………………
………………
vii. The right hepatocardiac channel
…………………………………………………………………………………
…………………………………………………………………………………
………………

22
viii. Left subcardinal vein
…………………………………………………………………………………
…………………………………………………………………………………
………………
ix. Allantois
…………………………………………………………………………………
…………………………………………………………………………………
………………
x. Ventral mesogastrium
……………………………………………………………………………………
……………………………………………………………………………………
…………

23
SECTION D 45 Marks

Label the following diagrams

1.

1. __________________________________

2. __________________________________

3. __________________________________

4. __________________________________

5. __________________________________

6. __________________________________

7. __________________________________

8. __________________________________

24
2.

1. _____________________________________

2. _____________________________________

3. _____________________________________

4. _____________________________________

5. _____________________________________

6. _____________________________________

7. _____________________________________

8. _____________________________________

9. _____________________________________

10. _____________________________________

25
3.

1. ________________________________

2. ________________________________

3. ________________________________

4. ________________________________

5. ________________________________

6. ________________________________

7. _________________________________

8. _________________________________

9. _________________________________

10. __________________________________

26
4.

5. Draw a typical spinal nerve

27
---------------------------------------------------------------------------------------
END OF EXAM

28
1. What does the term eversion mean?
A. Dorsi-flexion of the foot
B. Turning of the foot to make the sole face medially
C. Plantar-flexion of the foot
D. Turning of foot to make its sole face laterally
2. Which of the following terms best describes the movement of the thumb
towards the rest of the fingers?
A. Protraction
B. Pronation
C. Opposition
D. Retraction
3. What does the term „parfocal‟ mean?
A. The resolving power of an objective lens
B. Focusing your specimen using high magnification
C. Changing the objectives without refocusing
D. Using the coarse adjusting knob after moving from x10 to x40
objective

4. Which of the following is correct regarding the terms „resolving power‟?


A. The ocular lens determines the resolving power of a microscope
B. It is the largest distance between two particles at which they appear as
separate objects
C. It is the ability of the microscope to magnify many objects at the same
time
D. The objective with the high magnification has a high resolving power

5. Which of the following statements rightly describes a plane?


A. It is a solid line that is drawn on the body of a subject
B. It is the cut surface of the body.
C. The coronal and the midsagittal sections are examples
D. An imaginary flat surface dividing the body

6. Which of the following organelles are abundant in fibroblasts?


A. Free ribosomes
B. Proteasomes
C. Rough endoplasmic reticulum
D. Golgi apparatus

7. ……………………………… are numerous in highly metabolic cells.


A. Ribosomes
B. Smooth endoplasmic reticulum
C. Proteasomes
D. Mitochondria

8. Which statement is true about the plasma membrane?


A. It is made up of a single layer of phospholipids
B. The phospholipids have nonpolar heads
C. The phospholipids have polar fatty acid tails
D. The phospholipid molecules are amphipathic
9. Concerning the cell cycle
A. In G1 phase the cells are resting
B. In M phase, organelles mature
C. In G0 phase DNA is duplicate
D. In G2 karyokinesis occurs

10.Regarding the cell, which of the following is not true?


A. Neutrophils contain zymogenic granules
B. Macrophages have numerous lysosomes
C. Skeletal muscle cells have a prominent nucleolus
D. Basophils compliment mast cell in terms of function
11.Which of the following structures give the nucleus its structural support
framework?
A. Laminins
B. Microfilaments
C. Actins
D. Keratins

12.Which one of the following statements regarding cytoskeletons is correct?


A. The intermediate filaments are the smallest form of cytoskeletons
B. Microfilaments anchor in position some plasma membrane proteins
C. A pair of centrosomes make a centriole
D. Basal bodies in a cilium are made of nine pairs of microtubules
13.Which of the following epithelium lines the cornea of the eye?
A. Simple squamous epithelium
B. Pseudostratified columnar ciliated epithelium
C. Stratified squamous non-keratinized epithelium
D. Stratified columnar epithelium

14.Which of the following epithelium lines the pericardial cavity?


A. Simple squamous epithelium
B. Pseudostratified columnar ciliated epithelium
C. Endothelium
D. Simple columnar epithelium

15.Which of the following glands release their secretory products by the


apocrine mode of secretion?
A. Mammary
B. Sebaceous
C. Eccrine sweat glands
D. Pituitary

16.With regard to the lining of the ileum, the following are true
except………..
A. It is lined by a simple columnar epithelium
B. It has multicellular glandular structures, goblet cells
C. It has simple straight tubular glands, crypts of lieberkuhn
D. Paneth cells are present at the base of the intestinal glands

17.Brunner‟s glands are…………………………………..


A. Simple branched acinar glands
B. Simple branched tubular glands
C. Compound branched tubular glands
D. Compound tubulo-acinar glands

18.Which of the following is a follicular endocrine gland?


A. Pituitary
B. Thyroid
C. Pancreas
D. Adrenal

19.Which of the following cells have receptors for immunoglobulin E on their


surface?
A. Plasma
B. T-lymphocytes
C. Basophils
D. B-lymphocytes

20.Concerning the perichondrium, which of the following statements is


correct?
A. It consists of collagen type II fibres
B. It has an inner cell layer consisting of chondrocytes
C. It makes the outer covering of fibrocartilage
D. It has progenitor cells which become chondroblasts

21.Cartilage found in the external acoustic meatus is characterised by……


A. Cell nests of not more than two chondrocytes
B. Absence of perichondrium
C. Collagen type II fibres only
D. The same type of cartilage as that in the pubic symphysis

22.With cartilage regard to epiphyseal, which of the following is true?


A. It is fibrocartilage
B. Its arrest has no effect on growth of long bones
C. It is radiolucent
D. Its resting stage is characterised by osteocytes

23.Which of the following statements is correct about cartilage?


A. The territorial matrix is not the matrix that immediately surrounds the
lacuna
B. The interteritorrial matrix has no staining differences with the territorial
matrix
C. The territorial matrix is rich in glycosaminoglycans (GAGs)
D. Collagen fibres are less abundant in the interterritorial matrix
24.Compact bone is characterised by one of the following;
A. Osteons, also known as Harvesian canals
B. A cement line surrounding each Harvesian system
C. Osteocytes actively secreting extracellular matrix
D. Interconnected trabeculae of bone

25.One of the following is not true concerning woven bone.


A. It is the first type of bone to appear in embryonic life
B. It is replaced by lamellar bone in adult life
C. In adults it is found near sutures of the skull
D. It has same mineral content as lamellar bone

26.In intramembranous ossification,


A. Mesenchymal cells differentiate into osteoprogenitor cells
B. There is replacement of hyaline cartilage by bone tissue
C. In cranial flat bones, the bone in the centre forms faster than the
periphery
D. Osteocytes lose their contact with each other once bone has developed

27.Which of the following organs are supported by reticular tissue?


A. Spleen
B. Thymus
C. Brain
D. Tongue

28.Which of the following is correct regarding the trapezius muscle?


A. It is a flat diamond shaped muscle
B. It is innervated by the dorsal scapular nerve
C. It sends its proprioceptive fibres via C3 and C4
D. It makes up the superficial group of muscles of the back

29.To which of the following muscles of the back is the thoracodorsal nerve
immediately deep?
A. Latissmus
B. Trapezius
C. Rhomboid major
D. Rhomboid minor

30.Which of the following correctly describes the pectoralis major muscle?


A. It is made up of two parts, the clavicular and the sternocostal parts.
B. It is superficially covered by the clavipectoral fascia
C. It divides the axillary artery into the first, second and third parts
D. It is innervated by the lateral and medial pectoral nerves.

31.With regard to the vagi nerves, which statement is True?


A. Left vagus is anterior to the phrenic nerve
B. Left recurrent laryngeal hook around the left subclavian artery
C. Right recurrent laryngeal, hook around the ligamentum arteriosum
D. Left vagus passes posterior to the root of the left lung.

32.Where does the right sympathetic chain cross the diaphragm as it enters the
abdomen?
A. Through the aortic hiatus
B. Posterior to the diaphragm
C. Through the inferior vena caval opening
D. Through the oesophageal opening

33.Which of the following is correct concerning coronary vessels?


A. The right coronary artery always supplies the sinuatrio node
B. The coronary sinus is the only vessel that drains the heart walls
C. The coronary veins drain into the inferior vena cava
D. The left coronary artery partly supplies the right ventricle

34.Which of the following is NOT a characteristic of the pericardium?


A. It has a serous part
B. Pericarditis may cause accumulation of fluids in the pericardial cavity
C. The visceral pericardium is the inner most part
D. The fibrous pericardium is supplied by the vagus nerve

35.Which of the following organs is likely to be compressed in a patient with


an abnormally dilated left atrium?
A. Esophagus
B. Trachea
C. Thoracic duct
D. Azygos vein

36.In the posterior triangle of the neck,


A. The scalenusmedius is superior to the levator scapulae
B. The trunks of the brachial plexus are in between the scalenus anterior
and medius muscles
C. The semi-spinaliscapitis is inferior to the splenius capitis
D. The accessory nerve emerges below the midpoint of the posterior
border of the sternocleidoastoid muscle.

37.Mr Munalula comes to the hospital with a “swelling” on the left upper
lateral part of his back. On examination, you find that it is a „winged‟ left
scapula. Which of the following nerves is likely to have been affected?
A. Dorsal scapular
B. Accessory
C. Thoracodorsal
D. Long thoracic

38.You have a patient with a stab wound immediately above the medial third
of the left clavicle. Which of the following are you worried of?
A. Injury to the trachea
B. Pneumothorax
C. Perforation of the superior vena cava
D. Injury of the trunks of the brachial plexus

39.Which of the following is an abnormal constriction of the esophagus?


A. Diaphragmatic
B. Bronchial
C. Tracheal
D. Pharyngo-esophageal
40.What is the order of testicular coverings from superficial going deep.
A. Skin, dartos muscle, cremateric fascia, external spermatic fascia then
internal spermatic fascia
B. Skin, cremateric fascia, dartos muscle, external spermatic fascia then
internal spermatic fascia
C. Skin, dartos muscle, internal spermatic fascia cremateric fascia,
external spermatic fascia then
D. Skin, dartos muscle, external spermatic fascia, cremateric fascia then
internal spermatic fascia

41.Which of the following structures do you expect to find on the skin of a


premature baby?
A. Vellus
B. Terminal hair
C. Lanugo
D. Coarse hair

42.Concerning skin glands,


A. A seven months old fetus has functional Sebaceous glands
B. In women, eccrine sweat glands are affected by hormones
C. Apocrine gland are active from birth
D. Eccrine sweat glands are straight tubular glands

43.Moses twists Maria‟s forearm as they were playing. Which of the following
sensory structures of her skin were able to sense the twist?
A. Pacinian corpuscles
B. Krause end bulbs
C. Ruffini corpuscles
D. Meissner corpuscles

44.Which statement is FALSE regarding the heart?


A. The apex of the heart points anteriorly, inferiorly to the left
B. The apex of the heart is completed formed by the left ventricle
C. The base of the heart lies on the inferior aspect of the heart
D. The heart is obliquely oriented with reference to the median plane
45.Which of the following rightly describes the heart beat?
A. Ventricular systole proceeds atrial systole
B. The sinuatrial node depends on the brain to generate impulses
C. The apex of the heart is the last one to contract in one beat
D. Rise in ventricular blood volume closes the mitral valve

46.Regarding the lymphatic vessels,


A. The cisterns chyli lies anterior to the body of L1 on the right side and
posterior the abdominal aorta
B. Thoracic duct enters the thorax through the esophageal hiatus
C. The thoracic duct terminates at the union of the right and left
brachiocephalic veins
D. The cisterna chyli has colorless lymph

47.Which of the following is not one of the main vessels that drain into the
cisterna chyli?
A. Left lumbar trunk
B. Right lumbar trunk
C. Posterior intercostal trunk
D. Intestinal trunk

48.The mediastinal surface of each lung has a cardiac impression. Which of


the following statements correctly describes this impression?
A. On the right lung, the right ventricle forms the shallow impression
B. On the right lung, the deeper impression is produced by the right
atrium
C. On the left lung, a deeper impression is produced by the left atrium
D. On the left lung, the left ventricle produces the deeper impression

49.Which of the following is correct concerning cardiac innervation?


A. The superficial cardiac plexus is formed by fibres from all the cervical
sympathetic ganglia
B. The superficial cardiac plexus is formed by fibres from all the cervical
sympathetic ganglia and the recurrent laryngeal nerve
C. The superficial cardiac plexus is formed by fibres from the left superior
cervical sympathetic ganglion and the inferior cardiac branch of the
left vagus
D. The superficial cardiac plexus is formed by fibres from the right
superior cervical sympathetic ganglion and the inferior cardiac branch
of the right vagus

50.Which of the following is not true regarding the kidneys?


A. They are retroperitoneal
B. The renal sinuses separate the renal pyramids
C. The Gerota‟s fascia surrounds the fatty capsule
D. The renal corpuscles are found in the cortex

51.For a molecule to move from the blood stream to the Bowman‟s space to
become the glomerular filtrate, it passes through the following structures
except?
A. Podocytes
B. Filtration slit diaphragms
C. Endothelium
D. Fused basal laminae of capillaries and visceral layer of Bowman‟s
capsule

52.Concerning functions of the Juxtaglomerular Apparatus (JGA) all of the


following are correct EXCEPT
A. It produces rennin
B. It produces angiotensin
C. Involved in autoregulation of glomerular filtration rate
D. Located in the distal proximal convoluted tubule

53.Which of the following structures are not a characteristic of the visceral


part of the Bowman‟s capsule?
A. Podocytes
B. Primary processes
C. Pedicels
D. Vascular pole
54.Which of the following cells in the kidneys are involved in the process of
red blood cell production?
A. Mesangial cells
B. Juxtaglomerular granular cells
C. Extraglomerular mesangial cells
D. Fibroblasts

55.Which of these is not a function of the kidneys?


A. Regulation of acid-base balance
B. Secretion of angiotensin I
C. Activation of Vitamin D
D. Balance of water and electrolytes

56.Which of the following structures are not found in the renal corpuscle?
A. Podocytes
B. Capillaries
C. Macula densa
D. Mesangial cells

57.Concerning the structure of the male reproductive system,


A. The urethra passes through the corpus cavernosa
B. The penis has paired corporaspongiosa
C. The glans extends from the corpus spongiosum
D. Erectile tissue of the penis is made up of skeletal muscles

58.Which of the following structures have a lot of stereocilia on their


epithelium?
A. Epididymis
B. Ductus deferens
C. Prostatic urethra
D. Ejaculatory duct

59.Which of the following is incorrect regarding Sertoli cells?


A. They extend from the basal lamina to the luminal surface
B. The spermatids are invaginated into the Sertoli cells
C. They produce follicle stimulating hormone (FSH)
D. They phagocytize excess cytoplasm from the spermatids

60.Which statement is true concerning seminal vesicles?


A. They produce 30% of the ejaculate
B. The fructose in its secretion coagulates the semen after ejuclation
C. The prostaglandins in its secretion stimulate activity in the female
D. They store spermatozoa

61.What is the name of the capsule that surrounds the testes


A. tunica albuginea
B. tunica adventitia
C. tunica media
D. tunica vaginalis

62.Maturation of spermatozoa occurs in the


A. Seminiferous tubules
B. Epididymis
C. Rete testis
D. Vas deferens

63.Which of the following cells are not found in the interstitial tissue of the
testes
A. Fibroblasts
B. Sertoli
C. Myoid
D. Leydig

64.Which of the following cells of the testes produce the hormone


testosterone?
A. Leydig cells
B. Sertoli cells
C. Myoid cells
D. Mast cells

65.During penile erection, which nervous system causes vasodilation of blood


vessels of the penis?
A. Parasympathetic
B. Sympathetic
C. Somatic
D. Both sympathetic and parasympathetic

66.Which of the following is correct?


A. Luteinizing hormone stimulates Sertoli cells to release testosterone
B. Follicle stimulating hormone (FSH) stimulate Leydig cells of the
interstitium
C. Testosterone stimulate Leydig cells in the seminiferous tubules
D. Inhibin produced by Sertoli sells causes reduction in FSH

67.Which of the following organs is considered as a primary sex organ in


males?
A. Penis
B. Testis
C. Scrotum
D. Prostate gland

68.Concerning seminiferous tubules of the testes


A. Converge to become epididymis
B. Lined by simple cuboidal epithelium
C. Basement membrane covered by skeletal muscle fibres
D. They produce spermatozoa

69.Concerning the Vas deferens


A. Enlarged portion is called ampulla
B. Lined by simple columnar epithelium with stereocilia
C. Sperms are stored in the distal portion
D. Enters abdominal cavity via pubic arch

70.Which of the following hormones is responsible for ovulation


A. Progesterone
B. Luteinizing hormone
C. Follicle stimulating hormone
D. Estrogen

71.What gland in males is homologous to greater vestibular gland in females


A. Prostate gland
B. Bartholin‟s gland
C. Cowper‟s gland
D. Urethral glands
72.Female reproductive system
A. The epithelial cells of the uterus have microvilli
B. Uterine tubes epithelial cells are ciliated
C. The uterine glands are simple tubular
D. During menstruation, pars functionalis of the endometrium is not lost

73.In which of the following structures does fertilization occur?


A. Fimbriae
B. Infundibulum
C. Ampulla
D. Isthmus

74.Which of the following statements is false concerning oogenesis


A. Ovum is formed in the ovary from oogonia
B. Primary oocyte and its follicular cells constitute the primordial follicle
C. Primary oocyte begins its first meiotic division before birth
D. Oogonia remains dormant in the ovary until puberty

75.Concerning secondary oocyte at ovulation


A. The secondary oocyte is surrounded by the zonapellucida and corona
radiata
B. The secondary oocyte is at prophase II
C. The secondary oocyte has layers of follicular cells called Theca
D. The secondary oocyte has completed its second meiotic division

76.Which of the following structures is not part of the external genitalia?


A. Clitoris
B. Labia minora
C. Vestibule
D. Cervix

77.Which of the following components plays the most active role in invading
the endometrium during blastocyst implantation?
A. Epiblast
B. Syncytiotrophoblast
C. Hypoblast
D. Extraembryonic somatic mesoderm

78.Between which two layers is the extraembryonic mesoderm located?


A. Epiblast and hypoblast
B. Syncytiotrophoblast and cytotrophoblast
C. Exocoelomic membrane and syncytiotrophoblast
D. Exocoelomic membrane and Cytotrophoblast

79.During week 2 of development, the embryoblast receives its nutrients via


A. Diffusion
B. Osmosis
C. Yolk sac nourishment
D. Fetal capillaries

80.The prechordal plate marks the site of thefuture


A. Umbilical cord
B. Heart
C. Mouth
D. Nose

81.Which of the following are components of the definitive chorion?


A. Extraembryonic somatic mesoderm and epiblast
B. Extraembryonic somatic mesoderm and cytotrophoblast
C. Extraembryonic somatic mesoderm and syncytiotrophoblast
D. Extraembryonic somatic mesoderm, cytotrophoblast, and
syncytiotrophoblast

82.The first indication of gastrulation in the embryo is


A. Formation of the primitive streak
B. Formation of the notochord
C. Formation of the neural tube
D. Formation of extraembryonic mesoderm
83.Which of the following statements concerning the corpus luteum is true?
A. It produces LH.
B. It produces FSH.
C. It derives its granulosa luteal cells from the theca externa.
D. It becomes the corpus albicans.

84. LH exerts which one of the following physiological effects?


A. It triggers completion of the second meiotic division by secondary
oocytes.
B. It triggers ovulation.
C. It suppresses release of estrogens.
D. It induces primary follicles to become secondary follicles.

85.The basal layer of the uterine endometrium


A. Is sloughed during menstruation.
B. Has no glands.
C. Is supplied by coiled arteries.
D. Is supplied by straight arteries.

86.Which of the following statements concerning adrenal parenchymal cells is


true?
A. Those of the zona fasciculata produce androgens.
B. Those of the adrenal medulla produce epinephrine and norepinephrine.
C. Those of the zona glomerulosa produce glucocorticoids.
D. Those of the cortex contain numerous secretory granules.

87.Characteristics of pinealocytes include which one of the following?


A. They produce melatonin.
B. They resemble astrocytes.
C. They contain calcified concretions of unknown function.
D. They act as postganglionic sympathetic cells.

88.Prolactin is synthesized and secreted by which of the following cells?


A. Acidophils in the pars distalis
B. Basophils in the pars tuberalis
C. Somatotrophs in the pars distalis
D. Basophils in the pars intermedia
89.ACTH is produced by which of the following cells?
A. Chromophobes in the pars distalis
B. Neurosecretory cells in the medianeminence
C. Basophils in the pars distalis
D. Neurons of the paraventricular nucleusin the hypothalamus

90.The histological appearance of a thyroid gland being stimulated by TSH


would show which of the following?
A. Decreased numbers of follicular cells
B. Increased numbers of parafollicular cells
C. Column-shaped follicular cells
D. An abundance of colloid in the lumen of the follicle

91.A 40-year- old woman is diagnosed with Graves disease. Which of the
following characteristics would be associated with her condition?
A. Inadequate levels of iodine in her diet
B. Weight gain
C. Flattened thyroid follicular cells
D. Excessive production of thyroid hormones

92.Loss of patella reflex and cutaneous sensation on the anteromedial side of


leg indicate damage to spinal nerve:
A. L4
B. L5
C. S2
D. S3
93.Which of the following muscles is not a medial rotator of the leg?
A. Semitendinosus
B. Semimebranosus
C. Biceps femoris
D. Gracilis

94.Mulenga sustained an injury in the popliteal fossa injuring the tibial nerve.
This might result in:
A. Loss of eversion
B. Loss of sensation between
C. Inability to stand on one‟s toes
D. Foot drop
95.Which of the following is correct about the fibular artery:
A. Supplies the muscles of the anterior compartment of the leg
B. Passes anterior to the interosseous membrane
C. Usually becomes the dorsalis pedis
D. Courses through the deep posterior compartment of the leg
96.A tight plaster cast that exerted pressure on the head and neck of the fibula
might result in loss of:
A. Eversion
B. Plantar-flexion
C. Inversion
D. Flexion of the hallux
97.Where would you feel for the pulse of the dorsalis pedis artery?
A. Directly superficial to the intermediate cuneiform
B. Directly lateral to the tendon of the extensor digitorum longus
C. Directly in between the talus and the navicular bones
D. Directly medial to the extensor halluces longus
98.The blood supply to the posterior compartment muscles of the thigh is by:
A. Femoral artery
B. Popliteal artery
C. Profunda femoris artery
D. Obturator artery
99.The spinal cord segment that suppliescuteneous innervation to theblateral
side of the foot is:
A. L4
B. L5
C. S1
D. S2
100. The muscle that contracts to unlock the extended knee joint is the:
A. Popliteus
B. Plantaris
C. Medial head of gastrocnemius
D. Lateral head of gastrocnemius
SECTION B
Label the following diagrams

1.

1. __________________________________

2. __________________________________

3. __________________________________

4. __________________________________

5. __________________________________

6. __________________________________

7. __________________________________

8. __________________________________
2. Pectoral region

1. _____________________________________

2. _____________________________________

3. _____________________________________

4. _____________________________________

5. _____________________________________

6. _____________________________________

7. _____________________________________

8. _____________________________________

9. _____________________________________

10. _____________________________________
3.

1. ________________________________

2. ________________________________

3. ________________________________

4. ________________________________

5. ________________________________

6. ________________________________

7. _________________________________

8. _________________________________

9. _________________________________

10. __________________________________
4.

1. _________________________________

2. _________________________________

3. __________________________________

4. _________________________________

5. _________________________________

6. __________________________________

7. _________________________________

8. _________________________________

9. __________________________________
rartery
SECTION A 110 MARKS

Select the one most appropriate answer from the alternatives A-D in each question, by
marking X on the answer sheet grid provided. There is no penalty for wrong answers.

1. Which of the following statements rightly describes a plane?


A. It is a solid line that is drawn on the body of a subject
B. It is the cut surface of the body.
C. The coronal and the midsagittal sections are examples
D. An imaginary flat surface dividing the body

2. What does the term ‘parfocal’ mean?


A. The resolving power of an objective lens
B. Focusing your specimen using high magnification
C. Changing the objectives without refocusing
D. Using the coarse adjusting knob after moving from x10 to x40 objective

3. Which of the following is correct regarding the terms ‘resolving power’?


A. The ocular lens determines the resolving power of a microscope
B. It is the largest distance between two particles at which they appear as separate
objects
C. It is the ability of the microscope to magnify many objects at the same time
D. The objective with the high magnification has a high resolving power

4. With regard to the histologic appearance of a cell, which of the following is not true?
A. Fibroblasts have an extensive network of rough endoplasmic reticulum and golgi
apparatus
B. Macrophages have numerous lysosomes
C. Skeletal muscle cells have a prominent nucleolus
D. Secretory epithelial cells have rough endoplasmic reticulum concentrated at the
basal part

5. Which of the following structures give the nucleus its structural support framework?
A. Lamins
B. Microfilaments
C. Actin
D. Keratins
6. Which one of the following statements regarding cytoskeletons is correct?
A. The intermediate filaments are the smallest form of cytoskeletons
B. Microfilaments anchor in position some plasma membrane proteins
C. A pair of centrosomes make a centriole
D. Basal bodies in a cilium are made of nine pairs of microtubules

7. Which of the following glands release their secretory products by the apocrine method
of secretion?
A. Mammary
B. Sebaceous
C. Eccrine sweat glands
D. Pituitary

8. With regard to the lining of the ileum, the following are true except………..
A. It is lined by a simple columnar epithelium
B. It has multicellular glandular structures, goblet cells
C. It has simple straight tubular glands, crypts of lieberkuhn
D. Paneth cells are present at the base of the intestinal glands

9. In what class are the Brunner’s glands?


A. Simple branched acinar glands
B. Simple branched tubular glands
C. Compound branched tubular glands
D. Compound tubulo-acinar glands

10. Which of the following glands is a follicular gland


A. Pituitary
B. Thyroid
C. Pancreas
D. Adrenal

11. Which of the following anatomical terms means ‘on different sides of the body’?
A. Sagittal
B. Ipsilateral
C. Contralateral
D. Lateral
12. Which of the following terms best describes the movement of the thumb towards the
rest of the fingers?
A. Protraction
B. Pronation
C. Opposition
D. Retraction

13. Which of the following organelles initiate the process of apoptosis?


A. Mitochondria
B. Peroxisomes
C. Smooth endoplasmic reticulum
D. Lysosomes

14. Which of the following cells have receptors for immunoglobulin E on their surface?
A. Plasma
B. T-lymphocytes
C. Basophils
D. B-lymphocytes

15. Concerning the perichondrium, which of the following statements is correct?


A. It consists of collagen type II fibres
B. It has an inner cell layer consisting of chondrocytes
C. It makes the outer covering of fibrocartilage
D. It has progenitor cells which become chondroblasts

16. The cartilage found in the external acoustic meatus is characterised by……
A. Cell nests of not more than two chondrocytes
B. Absence of perichondrium
C. Collagen type II fibres only
D. The same type of cartilage as that in the pubic symphysis

17. Which of the following statements is correct about cartilage?


A. The territorial matrix is not the matrix that immediately surrounds the lacuna
B. The interteritorrial has no staining differences with the territorial matrix
C. The territorial matrix is rich in glycosaminoglycans (GAGs)
D. Collagen fibres are less abundant in the interterritorial matrix
18. Compact bone is characterised by one of the following;
A. Osteons, also known as Harvesian canals
B. A cement line surrounding each Harvesian system
C. Osteoblasts actively secreting extracellular matrix
D. Interconnected trabeculae of bone

19. One of the following is not true concerning woven bone.


A. It is the first type of bone to appear in embryonic life
B. It is replaced by lamellar bone in adult life
C. In adults it is found near sutures of the skull
D. It has same mineral content as lamellar bone

20. In intramembranous ossification,


A. Mesenchymal cells differentiate into osteoprogenitor cells
B. There is replacement of hyaline cartilage by bone tissue
C. In cranial flat bones, the bone in the centre forms faster than the periphery
D. Osteocytes lose their contact with each other once bone has developed

21. Which of the following is NOT true concerning mitochondria?


A. They replicate independently from the cell
B. They are passed on via the sperm
C. Contain their own genetic material
D. Vary in morphology between cells

22. Which of the following statements is true concerning cell cycle?


A. G1 Phase is the longest phase
B. Germ cells divide by mitosis in the M-phase
C. All the phases make up the interphase
D. The G2/M check point is just before cell division

23. Which of the following organelles are responsible for the formation of the granules in
mast cells?
A. Golgi apparatus
B. Lysosomes
C. Rough endoplasmic reticulum
D. Smooth endoplasmic reticulum
24. ………………………………….degrade unwanted proteins in cells.
A. Proteasomes
B. Smooth endoplasmic reticulum
C. Lysosomes
D. Mitochondria

25. Concerning the cell cycle


A. In G1 phase the cells are resting
B. In the M phase, there is cellular growth
C. In G0, cells synthesize DNA
D. G2 cells have a double complement of DNA

26. The duplication of centrosomes during interphase takes place in the…..


A. M phase
B. S phase
C. G1 phase
D. G2 phase

27. Which statement is true about the plasma membrane?


A. Glycocalyx is located on the inner surface of the plasma membrane
B. The phospholipids have nonpolar heads
C. The phospholipids have polar fatty acid tails
D. Cholesterol increases fluidity of the membrane

28. Which of the following organs are supported by reticular tissue?


A. Spleen
B. Thymus
C. Brain
D. Tongue

29. Which of the following statements is true about cartilage:


A. Fibrocartilage contains both type I and type II collagen fibers
B. Hyaline cartilage contains mainly type I collagen
C. Elastic cartilage contains both type I and elastin fibers
D. Cartilage gives rise to bone

30. Articular cartilage is characterized by all of the following except:


A. It is hyaline cartilage
B. Undergoes appositional growth
C. Regenerates poorly
D. Has a poor blood supply
31. With regard to epiphyseal cartilage, which of the following is true?
A. It is fibrocartilage
B. Its arrest has no effect on growth of long bones
C. It is radiolucent
D. Its resting stage is characterised by osteocytes

32. What type of growth does the cartilage in the pubic symphysis undergo?
A. Appositional and interstitial
B. Appositional
C. Chondrogenesis
D. Interstitial

33. Which of the following epithelium lines the cornea of the eye?
A. Simple squamous epithelium
B. Pseudostratified columnar ciliated epithelium
C. Stratified squamous non-keratinized epithelium
D. Stratified columnar epithelium

34. Which of the following epithelium lines the vestibule of the nose?
A. Simple squamous epithelia
B. Pseudostratified columnar ciliated epithelium
C. Stratified squamous non-keratinized epithelium
D. Simple columnar epithelium

35. Which of the following epithelium lines the pericardial cavity?


A. Simple squamous epithelium
B. Pseudostratified columnar ciliated epithelium
C. Endothelium
D. Simple columnar epithelium

36. Which of the following epithelium lines capillaries?


A. Urothelia
B. Mesothelium
C. Endothelium
D. Simple columnar epithelium
37. Concerning epithelial tissue,
A. It originates from endoderm exclusively
B. It rests on basement membrane
C. It has an abundance of extracellular matrix
D. Glandular epithelia is not epithelial tissue

38. Which of the following statements about bone tissue is correct?


A. It is made up of organic components solely.
B. For it to be radio-opaque, at least 30% of its components must be inorganic
C. It increases its thickness when the weight it is bearing increases
D. An osteon is newly laid extracellular matrix before it is mineralized

39. Osteocytic cytoplasmic extensions are housed in …………….


A. Cristae
B. Howship lacunae
C. Cisternae
D. Canaliculi

40. Concerning bone tissue, which of the following is true?


A. It is avascular
B. Osteoblasts secret extracellular matrix
C. Osteoid and osteon are at different stages of maturation
D. Harvesian canals are synonymous with Volkmann’s canals

41. Decreased bone density is seen in;


A. Osteogenesis imperfecta
B. Osteoporosis
C. Osteosarcoma
D. Osteosclerosis

42. Hyperparathyroidism may result in


A. Heavy bones
B. Osteopetrosis
C. Rickets
D. Fractures
43. Which of the following cells of connective tissue are actively involved in allergic
reactions?
A. Eosinophils
B. Plasma cells
C. Neutrophils
D. Mast cells

44. The following are true concerning leukocytes except:


A. Are wandering cells of connective tissue
B. Can be phagocytic
C. Carry out their function in connective tissue
D. Proliferate in epithelial tissue

45. Which of the following statements is true?


A. Osteocytes are highly active cells
B. Osteoblasts when active are squamous
C. Osteoclasts are derived from the bone marrow
D. Osteoblasts are useful in bone resorption

46. Which of the following is correct regarding the trapezius muscle?


A. It is a flat diamond shaped muscle
B. It is innervated by the dorsal scapular nerve
C. It sends its proprioceptive fibres via C3 and C4
D. It makes up the superficial group of muscle of the back

47. To which of the following muscles of the back is the thoracodorsal nerve immediately
deep?
A. Latissmus
B. Trapezius
C. Rhomboid major
D. Rhomboid minor

48. Which of the following correctly describes the the pectoralis major muscle?
A. It is made up of two parts, the clavicular and the sternocostal parts.
B. It is superficially covered by the clavipectoral fascia
C. It divides the axillary artery into the first, second and third parts
D. It is not pierced by the medial pectoral nerve.
49. With regard to the vagi nerves, which statement is True?
A. The left one is anterior to the phrenic nerve
B. The branch from the left one, recurrent laryngeal, hook around the left subclavian
artery
C. The branch from the right one, recurrent laryngeal, hook around the ligamentum
arteriosum
D. The left vagus passes posterior to the root of the left lung.

50. Where does the right sympathetic chain cross the diaphragm as it enter the abdomen?
A. Through the aortic hiatus
B. Behind the diaphragm
C. Through the inferior vena caval opening
D. Through the oesophageal opening

51. Which of the following is correct about coronary vessels?


A. The right coronary artery always supplies the sinuatrio node
B. The coronary sinus is the only vessel that drains the heart walls
C. The coronary veins drain into the inferior vena cava
D. The left coronary artery partly supplies the right ventricle

52. Which of the following is not a characteristic of the pericardium?


A. It has a serous part
B. Pericarditis may cause accumulation of fluids in the pericardial cavity
C. The visceral pericardium is the inner most part
D. The fibrous pericardium is supplied by the vagus nerve

53. Which of the following organs is likely to be compressed in a patient with an


abnormally dilated left atrium?
A. Esophagus
B. Trachea
C. Thoracic duct
D. Azygos vein

54. In the posterior triangle of the neck,


A. The scalenusmedius is superior to the levator scapulae
B. The trunks of the brachial plexus are in between the scalenus anterior and
mediusmuscles
C. The semi-spinaliscapitis is inferior to the splenius capitis
D. The accessory nerve emerges below the midpoint of the posterior border of the
sternocleidoastoid muscle.
55. Mr Munalula comes to the hospital with “swelling” on the left upper lateral part of his
back. On examination, you find that it is a ‘winged’ left scapula. Which of the following
nerves is likely to have been affected?
A. Dorsal scapular
B. Accessory
C. Thoracodorsal
D. Long thoracic

56. You have a patient with a stab wound immediately above the medial third of the left
clavicle. Which of the following are you worried of?
A. Injury to the trachea
B. Pneumothorax
C. Perforation of the superior vena cava
D. Injury of the trunks of the brachial plexus

57. A patient with a tumour of the left lung compressing on the esophagus causing him to
be vomiting after meals. A barium swallow x-ray is done. Which of the following is
most likely to be an abnormal constriction?
A. Diaphragmatic
B. Bronchial
C. Tracheal
D. Pharyngeoesophageal junction

58. You are the urologist in theatre carrying out orchidectomy(surgical removal of testes).
What layers are you going to cut through (in order) to get to the tunica vaginalis?
A. Skin, dartos muscle, cremateric fascia, external spermatic fascia then internal
spermatic fascia
B. Skin, cremateric fascia, dartos muscle, external spermatic fascia then internal
spermatic fascia
C. Skin, dartos muscle, internal spermatic fascia cremateric fascia, external
spermatic fascia then
D. Skin, dartos muscle, external spermatic fascia, cremateric fascia then internal
spermatic fascia

59. Which of the following structures do you expect to find on the skin of a premature
baby?
A. Vellus
B. Terminal hair
C. Lanugo
D. Coarse hair
60. Concerning skin glands,
A. A seven months old fetus has functional Sebaceous glands
B. Apocrine sweat glands are numerous on the skin of the nose
C. Sebaceous glands occur in the palms
D. Eccrine sweat glands are straight tubular glands

61. Which of the following is a characteristic of an eccrine sweat gland


A. The secretory part is characterised by columnar cells
B. Surrounding the basement membrane is a layer of myoepithelial cells
C. The duct is lined by a simple cuboidal epithelia
D. The duct does not alter the composition of sweat

62. Moses twists Maria’s forearm as they were playing. Which of the following sensory
structures of her skin were able to sense the twist?
A. Pacinian corpuscles
B. Krause end bulbs
C. Ruffini corpuscles
D. Meissner corpuscles

63. Which statement is FALSE regarding the heart?


A. The apex of the heart points anteriorly, inferiorly to the left
B. The apex of the heart is completed formed by the left ventricle
C. The base of the heart lies on the inferior aspect of the heart
D. The heart is obliquely oriented with reference to the median plane

64. Which of the following rightly describes the heart beat?


A. Ventricular systole proceeds atrial systole
B. The sinuatrial node depends on the brain to generate impulses
C. The apex of the heart is the last one to contract in one beat
D. Rise in ventricular blood volume closes the mitral valve

65. Regarding the lymphatic vessels,


A. The cisterns chyli lies anterior to the body of L1 on the right side and posterior the
abdominal aorta
B. Thoracic duct enters the thorax through the esophageal hiatus
C. The thoracic duct terminates at the union of the right and left brachiocephalic
veins
D. The cisterna chyli has colorless lymph
66. Which of the following is not one of the main vessels that drain into the cisterna chyli?
A. Left lumbar trunk
B. Right lumbar trunk
C. Posterior intercostal trunk
D. Intestinal trunk

67. The mediastinal surface of each lung has a cardiac impression. Which of the following
statements correctly describes this impression?
A. On the right lung, the right ventricle forms the shallow impression
B. On the right lung, the deeper impression is produced by the right atrium
C. On the left lung, a deeper impression is produced by the left atrium
D. On the left lung, the left ventricle produces the deeper impression

68. The sympathetic and parasympathetic systems accelerate and slow down the rate of
beating of the heart respectively. Which of the following is correct concerning cardiac
innervation?
A. The superficial cardiac plexus is formed by fibres from all the cervical sympathetic
ganglia
B. The superficial cardiac plexus is formed by fibres from all the cervical sympathetic
ganglia and the recurrent laryngeal nerve
C. The superficial cardiac plexus is formed by fibres from the left superior cervical
sympathetic ganglion and the inferior cardiac branch of the left vagus
D. The superficial cardiac plexus is formed by fibres from the right superior cervical
sympathetic ganglion and the inferior cardiac branch of the right vagus

69. Which of the following is not true regarding the kidneys?


A. They are retroperitoneal
B. The renal sinuses separate the renal pyramids
C. The Gerota’s fascia surrounds the fatty capsule
D. The renal corpuscles are found in the cortex

70. For a molecule to move from the blood stream to the Bowman’s space to become the
glomerular filtrate, it passes through the following structures except?
A. Podocytes
B. Filtration slit diaphragms
C. Endothelium
D. Fused basal laminae of capillaries and visceral layer of Bowman’s capsule
71. Concerning functions of the Juxtaglomerular Apparatus (JGA) all of the following are
correct EXCEPT
A. It produces rennin
B. It produces angiotensin
C. Involved in autoregulation of glomerular filtration rate
D. Located in the distal proximal convoluted tubule

72. Which of the following structures are not a characteristic of the visceral part of the
Bowman’s capsule?
A. Podocytes
B. Primary processes
C. Pedicels
D. Vascular pole

73. Which of the following cells in the kidneys are involved in the process of red blood cell
production?
A. Mesangial cells
B. Juxtaglomerular granular cells
C. Extraglomerularmesangial cells
D. Fibroblasts

74. Which of these is not a function of the kidneys?


A. Regulation of acid-base balance
B. Secretion of angiotensin I
C. Activation of Vitamin D
D. Balance of water and electrolytes

75. Which of the following structures are not found in the renal corpuscle?
A. Podocytes
B. Capillaries
C. Macula densa
D. Mesangial cells

76. Concerning the structure of the male reproductive system,


A. The urethra passes through the corpus cavernosa
B. The penis has paired corporaspongiosa
C. The glans extends from the corpus spongiosum
D. Erectile tissue of the penis is made up of skeletal muscles
77. Which of the following structures have a lot of stereocilia on their epithelium?
A. Epididymis
B. Ductus deferens
C. Prostatic urethra
D. Ejaculatory duct

78. Which of the following is incorrect regarding Sertoli cells?


A. They extend from the basal lamina to the luminal surface
B. The spermatids are invaginated into the Sertoli cells
C. They produce follicle stimulating hormone (FSH)
D. They phagocytize excess cytoplasm from the spermatids

79. Which statement is true concerning seminal vesicles?


A. They produce 30% of the ejaculate
B. The fructose in its secretion coagulates the semen after ejuclation
C. The prostaglandins in its secretion stimulate activity in the female
D. They store spermatozoa

80. What is the name of the capsule that surrounds the testes
A. tunica albuginea
B. tunica adventitia
C. tunica media
D. tunica vaginalis

81. Maturation of spermatozoa occurs in the


A. Seminiferous tubules
B. Epididymis
C. Rete testis
D. Vas deferens

82. Which of the following cells are not found in the interstitial tissue of the testes
A. Fibroblasts
B. Sertoli
C. Myoid
D. Leydig

83. Which of the following cells of the testes produce the hormone testosterone?
A. Leydig cells
B. Sertoli cells
C. Myoid cells
D. Mast cells
84. During penile erection, which nervous system causes vasodilation of blood vessels of
the penis?
A. Parasympathetic
B. Sympathetic
C. Somatic
D. Both sympathetic and parasympathetic

85. Which of the following is correct?


A. Luteinizing hormone stimulates Sertoli cells to release testosterone
B. Follicle stimulating hormone (FSH) stimulate Leydig cells of the interstitium
C. Testosterone stimulate Leydig cells in the seminiferous tubules
D. Inhibin produced by Sertoli sells causes reduction in FSH

86. Which of the following organs is considered as a primary sex organ in males?
A. Penis
B. Testis
C. Scrotum
D. Prostate gland

87. Concerning seminiferous tubules of the testes


A. Converge to become epididymis
B. Lined by simple cuboidal epithelium
C. Basement membrane covered by skeletal muscle fibres
D. They produce spermatozoa

88. Concerning the Vas deferens


A. Enlarged portion is called ampulla
B. Lined by simple columnar epithelium with stereocilia
C. Sperms are stored in the distal portion
D. Enters abdominal cavity via pubic arch

89. Which of the following hormones is responsible for ovulation


A. Progesterone
B. Luteinizing hormone
C. Follicle stimulating hormone
D. Estrogen

90. What gland in males is homologous to greater vestibular gland in females


A. Prostate gland
B. Bartholin’s gland
C. Cowper’s gland
D. Urethral glands
91. Female reproductive system
A. The epithelial cells of the uterus have microvilli
B. Uterine tubes epithelial cells are ciliated
C. The uterine glands are simple tubular
D. During menstruation, pars functionalis of the endometrium is not lost

92. In which of the following structures does fertilization occur?


A. Fimbriae
B. Infundibulum
C. Ampulla
D. Isthmus

93. Which of the following statements is false concerning oogenesis


A. Ovum is formed in the ovary from oogonia
B. Primary oocyte and its follicular cells constitute the primordial follicle
C. Primary oocyte begins its first meiotic division before birth
D. Oogonia remains dormant in the ovary until puberty

94. Concerning secondary oocyte at ovulation


A. The secondary oocyte is surrounded by the zonapellucida and corona radiata
B. The secondary oocyte is at prophase II
C. The secondary oocyte has layers of follicular cells called Theca
D. The secondary oocyte has completed its second meiotic division

95. Which of the following structures is not part of the external genitalia?
A. Clitoris
B. Labia minora
C. Vestibule
D. Cervix

96. Which of the following components plays the most active role in invading the
endometriumduring blastocyst implantation?
A. Epiblast
B. Syncytiotrophoblast
C. Hypoblast
D. Extraembryonic somatic mesoderm
97. Between which two layers is the extraembryonic mesoderm located?
A. Epiblast and hypoblast
B. Syncytiotrophoblast and cytotrophoblast
C. Exocoelomic membrane and syncytiotrophoblast
D. Exocoelomic membrane and Cytotrophoblast

98. During week 2 of development, the embryoblast receives its nutrients via
A. Diffusion
B. Osmosis
C. Yolk sac nourishment
D. Fetal capillaries

99. The prochordal plate marks the site of thefuture


A. Umbilical cord
B. Heart
C. Mouth
D. Nose

100. Which of the following are components of the definitive chorion?


A. Extraembryonic somatic mesoderm and epiblast
B. Extraembryonic somatic mesoderm and cytotrophoblast
C. Extraembryonic somatic mesoderm and syncytiotrophoblast
D. Extraembryonic somatic mesoderm, cytotrophoblast, and syncytiotrophoblast

101. The first indication of gastrulation in the embryo is


A. Formation of the primitive streak
B. Formation of the notochord
C. Formation of the neural tube
D. Formation of extraembryonic mesoderm

102. Which of the following statements concerning the corpus luteum is true?
A. It produces LH.
B. It produces FSH. PROGESTERO AND ESTOROGEN
C. It derives its granulosa luteal cells from the theca externa.
D. It becomes the corpus albicans.

103. LH exerts which one of the following physiological effects?


A. It triggers completion of the second meiotic division by secondary oocytes.
B. It triggers ovulation.
C. It suppresses release of estrogens.
D. It induces primary follicles to become secondary follicles.
104. The basal layer of the uterine endometrium
A. Is sloughed during menstruation.
B. Has no glands.
C. Is supplied by coiled arteries.
D. Is supplied by straight arteries.

105. Which of the following statements concerning adrenal parenchymal cells is true?
A. Those of the zona fasciculata produce androgens.
B. Those of the adrenal medulla produce epinephrine and norepinephrine.
C. Those of the zona glomerulosa produce glucocorticoids.
D. Those of the cortex contain numerous secretory granules.

106. Characteristics of pinealocytes include which one of the following?


A. They produce melatonin.
B. They resemble astrocytes.
C. They contain calcified concretions of unknown function.
D. They act as postganglionic sympathetic cells.

107. Prolactin is synthesized and secreted by which of the following cells?


A. Acidophils in the pars distalis
B. Basophils in the pars tuberalis
C. Somatotrophs in the pars distalis
D. Basophils in the pars intermedia

108. ACTH is produced by which of the following cells?


A. Chromophobes in the pars distalis
B. Neurosecretory cells in the medianeminence
C. Basophils in the pars distalis
D. Neurons of the paraventricular nucleusin the hypothalamus

109. The histological appearance of a thyroid gland being stimulated by TSH would show
which of the following?
A. Decreased numbers of follicular cells
B. Increased numbers of parafollicular cells
C. Column-shaped follicular cells
D. An abundance of colloid in the lumen of the follicle
110. A 40-year- old woman is diagnosed with Graves disease. Which of the following
characteristics would be associated with her condition?
A. Inadequate levels of iodine in her diet
B. Weight gain
C. Flattened thyroid follicular cells
D. Excessive production of thyroid hormones

SECTION B 30 MARKS

Short Answer Questions

1. Outline the at least five functions of the endothelium. Filtration, Gaseous exchange,
Pinocytosis, Diffusion, (5)
2. With an aid of a diagram, describe the conducting system of the heart. (5)
3. Give the boundaries of the inguinal canal and its contents. (5)
4. Outline the lymphatic drainage of the stomach. (5)
5. Briefly compare and contrast oogenesis and spermatogenesis (10)

SECTION C 22 Marks

Label the following diagrams below: label against the tags

1.

2.

3.

You might also like